Ubuntu 12.10 Vs Ati catalist 12.6

Postate qui per tutte le discussioni legate a Linux in generale.

Moderatore: Staff

Regole del forum
1) Citare sempre la versione di Slackware usata, la versione del Kernel e magari anche la versione della libreria coinvolta. Questi dati aiutano le persone che possono rispondere.
2) Per evitare confusione prego inserire in questo forum solo topic che riguardano appunto Gnu/Linux in genere, se l'argomento è specifico alla Slackware usate uno dei forum Slackware o Slackware64.
3) Leggere attentamente le risposte ricevute
4) Scrivere i messaggi con il colore di default, evitare altri colori.
5) Scrivere in Italiano o in Inglese, se possibile grammaticalmente corretto, evitate stili di scrittura poco chiari, quindi nessuna abbreviazione tipo telegramma o scrittura stile SMS o CHAT.
6) Appena registrati è consigliato presentarsi nel forum dedicato.

La non osservanza delle regole porta a provvedimenti di vari tipo da parte dello staff, in particolare la non osservanza della regola 5 porta alla cancellazione del post e alla segnalazione dell'utente. In caso di recidività l'utente rischia il ban temporaneo.
DarkNeo
Linux 4.x
Linux 4.x
Messaggi: 1071
Iscritto il: sab 4 giu 2005, 0:00
Nome Cognome: Stefano Arbitrio
Slackware: Slackware64 14
Kernel: 3.2.X
Desktop: KDE
Località: Matrix

Ubuntu 12.10 Vs Ati catalist 12.6

Messaggio da DarkNeo »

Ciao e buon anno a tutti, io sono rimasto ancora con i problemi del 2012 riguardante la scheda grafica ati radeon hd 4200 la quale è compatibile con la versione 12.6 dei driver ati proprietari. Ho installato ubuntu su vmware l'ho aggiornato ho installato, ho installato i pacchetti necessari:

Codice: Seleziona tutto

sudo apt-get install build-essential cdbs dh-make dkms execstack dh-modaliases linux-headers-generic
sudo apt-get install lib32gcc1
e ho seguito l'ormai famosa guida consigliata da 414N per effettuare il downgrade del serve X e per installare i driver:

Codice: Seleziona tutto

sudo add-apt-repository ppa:makson96/fglrx
sudo apt-get update
sudo apt-get upgrade
sudo apt-get install fglrx-legacy
ho riavviato X e niente monitor in bassa risoluzione e privo di icone. risultato

Codice: Seleziona tutto

root@ubuntu:~# fglrxinfo 
X Error of failed request:  BadRequest (invalid request code or no such operation)
  Major opcode of failed request:  136 (GLX)
  Minor opcode of failed request:  19 (X_GLXQueryServerString)
  Serial number of failed request:  12
  Current serial number in output stream:  12
Come al solito... Unica differenza è che in /etc/X11/ non cè xorg.conf

Codice: Seleziona tutto

root@ubuntu:~# ls -l  /etc/X11/
total 49708
drwxr-xr-x 2 root root     4096 Oct 17 17:00 app-defaults
-rw------- 1 root root 52518912 Jan 10 17:05 core
drwxr-xr-x 2 root root     4096 Oct 17 17:00 cursors
-rw-r--r-- 1 root root       18 Oct 17 17:00 default-display-manager
drwxr-xr-x 4 root root     4096 Oct 17 16:59 fonts
-rw-r--r-- 1 root root    17394 Dec  3  2009 rgb.txt
lrwxrwxrwx 1 root root       13 Jan 10 16:03 X -> /usr/bin/Xorg
drwxr-xr-x 3 root root     4096 Oct 17 17:00 xinit
drwxr-xr-x 2 root root     4096 Sep 28 00:52 xkb
-rwxr-xr-x 1 root root      709 Apr  1  2010 Xreset
drwxr-xr-x 2 root root     4096 Oct 17 16:59 Xreset.d
drwxr-xr-x 2 root root     4096 Oct 17 16:59 Xresources
-rwxr-xr-x 1 root root     3730 Mar 22  2012 Xsession
drwxr-xr-x 2 root root     4096 Jan 10 17:44 Xsession.d
-rw-r--r-- 1 root root      265 Jul  1  2008 Xsession.options
-rw-r--r-- 1 root root       13 Aug 15 11:43 XvMCConfig
-rw-r--r-- 1 root root      601 Oct 17 16:59 Xwrapper.config
l'unico errore che invece sta nel /var/log/Xorg.0.log è questo

Codice: Seleziona tutto

[    40.485] (EE) GLX error: Can not get required symbols.

Avatar utente
414N
Iper Master
Iper Master
Messaggi: 2922
Iscritto il: mer 13 feb 2008, 16:19
Slackware: 15.0
Kernel: 5.15.19
Desktop: KDE5
Località: Bulagna
Contatta:

Re: Ubuntu 12.10 Vs Ati catalist 12.6

Messaggio da 414N »

È inutile installare i driver proprietari su una macchina virtuale fornita da VMWare, visto che questi emula una scheda video diversa da quella fisica.

DarkNeo
Linux 4.x
Linux 4.x
Messaggi: 1071
Iscritto il: sab 4 giu 2005, 0:00
Nome Cognome: Stefano Arbitrio
Slackware: Slackware64 14
Kernel: 3.2.X
Desktop: KDE
Località: Matrix

Re: Ubuntu 12.10 Vs Ati catalist 12.6

Messaggio da DarkNeo »

Ah, non lo sapevo, comunque l'errore che ho nella partizione ubuntu nel /var/log/Xorg.0.log è questo:

Codice: Seleziona tutto

[    21.583] 
X.Org X Server 1.12.4
Release Date: 2012-08-27
[    21.583] X Protocol Version 11, Revision 0
[    21.583] Build Operating System: Linux 2.6.24-32-xen x86_64 Ubuntu
[    21.583] Current Operating System: Linux DarkNeo 3.5.0-17-generic #28-Ubuntu
 SMP Tue Oct 9 19:31:23 UTC 2012 x86_64
[    21.583] Kernel command line: BOOT_IMAGE=/boot/vmlinuz-3.5.0-17-generic root
=UUID=b6fdf641-f65d-4b5c-88fd-14cf062f7785 ro quiet splash vt.handoff=7
[    21.583] Build Date: 05 November 2012  10:00:21AM
[    21.583] xorg-server 3:1.12.4+git20121105-makson1~ppa2 (For technical suppor
t please see http://www.ubuntu.com/support) 
[    21.583] Current version of pixman: 0.26.0
[    21.583] 	Before reporting problems, check http://wiki.x.org
	to make sure that you have the latest version.
[    21.583] Markers: (--) probed, (**) from config file, (==) default setting,
	(++) from command line, (!!) notice, (II) informational,
	(WW) warning, (EE) error, (NI) not implemented, (??) unknown.
[    21.583] (==) Log file: "/var/log/Xorg.0.log", Time: Sun Jan 13 14:08:29 201
3
[    21.656] (==) Using config file: "/etc/X11/xorg.conf"
[    21.583] 
X.Org X Server 1.12.4
Release Date: 2012-08-27
[    21.583] X Protocol Version 11, Revision 0
[    21.583] Build Operating System: Linux 2.6.24-32-xen x86_64 Ubuntu
[    21.583] Current Operating System: Linux DarkNeo 3.5.0-17-generic #28-Ubuntu
 SMP Tue Oct 9 19:31:23 UTC 2012 x86_64
[    21.583] Kernel command line: BOOT_IMAGE=/boot/vmlinuz-3.5.0-17-generic root
=UUID=b6fdf641-f65d-4b5c-88fd-14cf062f7785 ro quiet splash vt.handoff=7
[    21.583] Build Date: 05 November 2012  10:00:21AM
[    21.583] xorg-server 3:1.12.4+git20121105-makson1~ppa2 (For technical suppor
t please see http://www.ubuntu.com/support) 
[    21.583] Current version of pixman: 0.26.0
[    21.583] 	Before reporting problems, check http://wiki.x.org
	to make sure that you have the latest version.
[    21.583] Markers: (--) probed, (**) from config file, (==) default setting,
	(++) from command line, (!!) notice, (II) informational,
	(WW) warning, (EE) error, (NI) not implemented, (??) unknown.
[    21.583] (==) Log file: "/var/log/Xorg.0.log", Time: Sun Jan 13 14:08:29 201
3
[    21.656] (==) Using config file: "/etc/X11/xorg.conf"
[    21.583] 
X.Org X Server 1.12.4
Release Date: 2012-08-27
[    21.583] X Protocol Version 11, Revision 0
[    21.583] Build Operating System: Linux 2.6.24-32-xen x86_64 Ubuntu
[    21.583] Current Operating System: Linux DarkNeo 3.5.0-17-generic #28-Ubuntu
 SMP Tue Oct 9 19:31:23 UTC 2012 x86_64
[    21.583] Kernel command line: BOOT_IMAGE=/boot/vmlinuz-3.5.0-17-generic root
=UUID=b6fdf641-f65d-4b5c-88fd-14cf062f7785 ro quiet splash vt.handoff=7
[    21.583] Build Date: 05 November 2012  10:00:21AM
[    21.583] xorg-server 3:1.12.4+git20121105-makson1~ppa2 (For technical suppor
t please see http://www.ubuntu.com/support) 
[    21.583] Current version of pixman: 0.26.0
[    21.583] 	Before reporting problems, check http://wiki.x.org
	to make sure that you have the latest version.
[    21.583] Markers: (--) probed, (**) from config file, (==) default setting,
	(++) from command line, (!!) notice, (II) informational,
	(WW) warning, (EE) error, (NI) not implemented, (??) unknown.
[    21.583] (==) Log file: "/var/log/Xorg.0.log", Time: Sun Jan 13 14:08:29 201
3
[    21.656] (==) Using config file: "/etc/X11/xorg.conf"
[    21.656] (==) Using system config directory "/usr/share/X11/xorg.conf.d"
[    21.689] (==) ServerLayout "aticonfig Layout"
[    21.689] (**) |-->Screen "aticonfig-Screen[0]-0" (0)
[    21.689] (**) |   |-->Monitor "aticonfig-Monitor[0]-0"
[    21.689] (**) |   |-->Device "aticonfig-Device[0]-0"
[    21.689] (==) Automatically adding devices
[    21.689] (==) Automatically enabling devices
[    21.735] (WW) The directory "/usr/share/fonts/X11/cyrillic" does not exist.
[    21.735] 	Entry deleted from font path.
[    21.735] (WW) The directory "/usr/share/fonts/X11/100dpi/" does not exist.
[    21.735] 	Entry deleted from font path.
[    21.735] (WW) The directory "/usr/share/fonts/X11/75dpi/" does not exist.
[    21.735] 	Entry deleted from font path.
[    21.735] (WW) The directory "/usr/share/fonts/X11/100dpi" does not exist.
[    21.735] 	Entry deleted from font path.
[    21.735] (WW) The directory "/usr/share/fonts/X11/75dpi" does not exist.
[    21.735] 	Entry deleted from font path.
[    21.735] (WW) The directory "/var/lib/defoma/x-ttcidfont-conf.d/dirs/TrueTyp
e" does not exist.
[    21.735] 	Entry deleted from font path.
[    21.735] (==) FontPath set to:
	/usr/share/fonts/X11/misc,
	/usr/share/fonts/X11/Type1,
	built-ins
[    21.735] (==) ModulePath set to "/usr/lib/x86_64-linux-gnu/xorg/extra-module
s,/usr/lib/xorg/extra-modules,/usr/lib/xorg/modules"
[    21.735] (II) The server relies on udev to provide the list of input devices
.
	If no devices become available, reconfigure udev or disable AutoAddDevic
es.
[    21.735] (II) Loader magic: 0x7f8e244fcb20
[    21.735] (II) Module ABI versions:
[    21.735] 	X.Org ANSI C Emulation: 0.4
[    21.735] 	X.Org Video Driver: 12.2
[    21.735] 	X.Org XInput driver : 16.0
[    21.735] 	X.Org Server Extension : 6.0
[    21.736] (--) PCI:*(0:1:5:0) 1002:9712:1025:0293 rev 0, Mem @ 0xd0000000/268
435456, 0xcfdf0000/65536, 0xcfe00000/1048576, I/O @ 0x00009000/256
[    21.736] (II) Open ACPI successful (/var/run/acpid.socket)
[    21.736] (II) "extmod" will be loaded by default.
[    21.736] (II) "dbe" will be loaded by default.
[    21.736] (II) "glx" will be loaded by default.
[    21.736] (II) "record" will be loaded by default.
[    21.736] (II) "dri" will be loaded by default.
[    21.736] (II) "dri2" will be loaded by default.
[    21.736] (II) LoadModule: "extmod"
[    21.750] (II) Loading /usr/lib/xorg/modules/extensions/libextmod.so
[    21.766] (II) Module extmod: vendor="X.Org Foundation"
[    21.766] 	compiled for 1.12.4, module version = 1.0.0
[    21.766] 	Module class: X.Org Server Extension
[    21.766] 	ABI class: X.Org Server Extension, version 6.0
[    21.766] (II) Loading extension MIT-SCREEN-SAVER
[    21.766] (II) Loading extension XFree86-VidModeExtension
[    21.766] (II) Loading extension XFree86-DGA
[    21.766] (II) Loading extension DPMS
[    21.766] (II) Loading extension XVideo
[    21.766] (II) Loading extension XVideo-MotionCompensation
[    21.766] (II) Loading extension X-Resource
[    21.766] (II) LoadModule: "dbe"
[    21.766] (II) Loading /usr/lib/xorg/modules/extensions/libdbe.so
[    21.767] (II) Module dbe: vendor="X.Org Foundation"
[    21.767] 	compiled for 1.12.4, module version = 1.0.0
[    21.767] 	Module class: X.Org Server Extension
[    21.767] 	ABI class: X.Org Server Extension, version 6.0
[    21.767] (II) Loading extension DOUBLE-BUFFER
[    21.767] (II) LoadModule: "glx"
[    21.767] (II) Loading /usr/lib/x86_64-linux-gnu/xorg/extra-modules/extra-mod
ules.dpkg-tmp/modules/extensions/libglx.so
[    21.802] (II) Module glx: vendor="Advanced Micro Devices, Inc."
[    21.802] 	compiled for 6.9.0, module version = 1.0.0
[    21.802] (II) Loading extension GLX
[    21.802] (II) LoadModule: "record"
[    21.803] (II) Loading /usr/lib/xorg/modules/extensions/librecord.so
[    21.803] (II) Module record: vendor="X.Org Foundation"
[    21.803] 	compiled for 1.12.4, module version = 1.13.0
[    21.803] 	Module class: X.Org Server Extension
[    21.803] 	ABI class: X.Org Server Extension, version 6.0
[    21.803] (II) Loading extension RECORD
[    21.803] (II) LoadModule: "dri"
[    21.804] (II) Loading /usr/lib/xorg/modules/extensions/libdri.so
[    21.818] (II) Module dri: vendor="X.Org Foundation"
[    21.818] 	compiled for 1.12.4, module version = 1.0.0
[    21.818] 	ABI class: X.Org Server Extension, version 6.0
[    21.818] (II) Loading extension XFree86-DRI
[    21.818] (II) LoadModule: "dri2"
[    21.819] (II) Loading /usr/lib/xorg/modules/extensions/libdri2.so
[    21.819] (II) Module dri2: vendor="X.Org Foundation"
[    21.819] 	compiled for 1.12.4, module version = 1.2.0
[    21.819] 	ABI class: X.Org Server Extension, version 6.0
[    21.819] (II) Loading extension DRI2
[    21.819] (II) LoadModule: "fglrx"
[    21.819] (II) Loading /usr/lib/x86_64-linux-gnu/xorg/extra-modules/extra-mod
ules.dpkg-tmp/modules/drivers/fglrx_drv.so
[    22.146] (II) Module fglrx: vendor="FireGL - ATI Technologies Inc."
[    22.146] 	compiled for 1.4.99.906, module version = 8.97.2
[    22.146] 	Module class: X.Org Video Driver
[    22.147] (II) Loading sub module "fglrxdrm"
[    22.147] (II) LoadModule: "fglrxdrm"
[    22.147] (II) Loading /usr/lib/x86_64-linux-gnu/xorg/extra-modules/extra-mod
ules.dpkg-tmp/modules/linux/libfglrxdrm.so
[    22.215] (II) Module fglrxdrm: vendor="FireGL - ATI Technologies Inc."
[    22.215] 	compiled for 1.4.99.906, module version = 8.97.2
[    22.215] (II) ATI Proprietary Linux Driver Version Identifier:8.97.2
[    22.215] (II) ATI Proprietary Linux Driver Release Identifier: 8.97.100.3 
[    22.215] (II) ATI Proprietary Linux Driver Build Date: Jul  3 2012 23:56:30
[    22.215] (++) using VT number 7

[    22.215] (WW) Falling back to old probe method for fglrx
[    22.349] (II) PCS database file /etc/ati/amdpcsdb not found
[    22.349] (II)   Creating PCS database from initial defaults instead
[    22.363] (--) Chipset Supported AMD Graphics Processor (0x9712) found
[    22.379] (WW) fglrx: No matching Device section for instance (BusID PCI:0@0:
17:0) found
[    22.379] (WW) fglrx: No matching Device section for instance (BusID PCI:0@0:
18:0) found
[    22.379] (WW) fglrx: No matching Device section for instance (BusID PCI:0@0:
18:1) found
[    22.379] (WW) fglrx: No matching Device section for instance (BusID PCI:0@0:
18:2) found
[    22.379] (WW) fglrx: No matching Device section for instance (BusID PCI:0@0:
19:0) found
[    22.379] (WW) fglrx: No matching Device section for instance (BusID PCI:0@0:
19:1) found
[    22.379] (WW) fglrx: No matching Device section for instance (BusID PCI:0@0:
19:2) found
[    22.379] (WW) fglrx: No matching Device section for instance (BusID PCI:0@0:
20:0) found
[    22.379] (WW) fglrx: No matching Device section for instance (BusID PCI:0@0:
20:2) found
[    22.379] (WW) fglrx: No matching Device section for instance (BusID PCI:0@0:
20:3) found
[    22.379] (WW) fglrx: No matching Device section for instance (BusID PCI:0@0:
20:4) found
[    22.379] (WW) fglrx: No matching Device section for instance (BusID PCI:0@1:
5:1) found
[    22.414] (II) AMD Video driver is running on a device belonging to a group t
argeted for this release
[    22.414] (II) AMD Video driver is signed
[    22.414] (II) fglrx(0): pEnt->device->identifier=0x7f8e25013c50
[    22.414] (II) fglrx(0): === [xdl_xs112_atiddxPreInit] === begin
[    22.414] (II) Loading sub module "vgahw"
[    22.414] (II) LoadModule: "vgahw"
[    22.452] (II) Loading /usr/lib/xorg/modules/libvgahw.so
[    22.471] (II) Module vgahw: vendor="X.Org Foundation"
[    22.471] 	compiled for 1.12.4, module version = 0.1.0
[    22.471] 	ABI class: X.Org Video Driver, version 12.2
[    22.471] (**) fglrx(0): Depth 24, (--) framebuffer bpp 32
[    22.471] (II) fglrx(0): Pixel depth = 24 bits stored in 4 bytes (32 bpp pixm
aps)
[    22.471] (==) fglrx(0): Default visual is TrueColor
[    22.471] (**) fglrx(0): Option "DPMS" "true"
[    22.471] (==) fglrx(0): RGB weight 888
[    22.471] (II) fglrx(0): Using 8 bits per RGB 
[    22.471] (==) fglrx(0): Buffer Tiling is ON
[    22.471] (II) Loading sub module "fglrxdrm"
[    22.471] (II) LoadModule: "fglrxdrm"
[    22.472] (II) Loading /usr/lib/x86_64-linux-gnu/xorg/extra-modules/extra-mod
ules.dpkg-tmp/modules/linux/libfglrxdrm.so
[    22.472] (II) Module fglrxdrm: vendor="FireGL - ATI Technologies Inc."
[    22.472] 	compiled for 1.4.99.906, module version = 8.97.2
[    22.474] ukiDynamicMajor: failed to open /proc/ati/major
[    22.474] ukiDynamicMajor: failed to open /proc/ati/major
[    22.474] (**) fglrx(0): NoAccel = NO
[    22.474] (**) fglrx(0): ATI 2D Acceleration Architecture enabled
[    22.474] (--) fglrx(0): Chipset: "ATI Mobility Radeon HD 4200 Series" (Chips
et = 0x9712)
[    22.474] (--) fglrx(0): (PciSubVendor = 0x1025, PciSubDevice = 0x0293)
[    22.474] (==) fglrx(0): board vendor info: third party graphics adapter - NO
T original ATI
[    22.474] (--) fglrx(0): Linear framebuffer (phys) at 0xd0000000
[    22.474] (--) fglrx(0): MMIO registers at 0xcfdf0000
[    22.474] (--) fglrx(0): I/O port at 0x00009000
[    22.474] (==) fglrx(0): ROM-BIOS at 0x000c0000
[    22.557] (II) fglrx(0): Primary V_BIOS segment is: 0xc000
[    22.587] (II) Loading sub module "vbe"
[    22.587] (II) LoadModule: "vbe"
[    22.587] (II) Loading /usr/lib/xorg/modules/libvbe.so
[    22.598] (II) Module vbe: vendor="X.Org Foundation"
[    22.598] 	compiled for 1.12.4, module version = 1.1.0
[    22.598] 	ABI class: X.Org Video Driver, version 12.2
[    22.610] (II) fglrx(0): VESA BIOS detected
[    22.610] (II) fglrx(0): VESA VBE Version 3.0
[    22.610] (II) fglrx(0): VESA VBE Total Mem: 16384 kB
[    22.610] (II) fglrx(0): VESA VBE OEM: ATI ATOMBIOS
[    22.610] (II) fglrx(0): VESA VBE OEM Software Rev: 10.94
[    22.610] (II) fglrx(0): VESA VBE OEM Vendor: (C) 1988-2005, ATI Technologies
 Inc. 
[    22.610] (II) fglrx(0): VESA VBE OEM Product: RS880M
[    22.610] (II) fglrx(0): VESA VBE OEM Product Rev: 01.00
[    22.619] (II) fglrx(0): ATI Video BIOS revision 9 or later detected
[    22.619] (--) fglrx(0): Video RAM: 262144 kByte, Type: DDR2
[    22.619] (II) fglrx(0): PCIE card detected
[    22.619] (--) fglrx(0): Using per-process page tables (PPPT) as GART.
[    22.619] (WW) fglrx(0): board is an unknown third party board, chipset is su
pported
[    22.619] (WW) fglrx(0): Hasn't establisted DRM connection
[    22.619] (II) fglrx(0): [FB] MC range(MCFBBase = 0xc0000000, MCFBSize = 0x10
000000)
[    22.619] (WW) fglrx(0): No DRM connection for driver fglrx.
[    22.619] (II) fglrx(0): RandR 1.2 support is enabled!
[    22.619] (II) fglrx(0): RandR 1.2 rotation support is enabled!
[    22.619] (==) fglrx(0): Center Mode is disabled 
[    22.619] (II) Loading sub module "fb"
[    22.619] (II) LoadModule: "fb"
[    22.620] (II) Loading /usr/lib/xorg/modules/libfb.so
[    22.649] (II) Module fb: vendor="X.Org Foundation"
[    22.649] 	compiled for 1.12.4, module version = 1.0.0
[    22.649] 	ABI class: X.Org ANSI C Emulation, version 0.4
[    22.649] (II) Loading sub module "ddc"
[    22.649] (II) LoadModule: "ddc"
[    22.649] (II) Module "ddc" already built-in
[    23.943] (II) fglrx(0): Output LVDS using monitor section aticonfig-Monitor[
0]-0
[    23.943] (II) fglrx(0): Output DFP1 has no monitor section
[    23.943] (II) fglrx(0): Output CRT1 has no monitor section
[    23.943] (II) Loading sub module "ddc"
[    23.943] (II) LoadModule: "ddc"
[    23.943] (II) Module "ddc" already built-in
[    23.943] (II) fglrx(0): Connected Display0: LVDS
[    23.943] (II) fglrx(0): Display0 EDID data ---------------------------
[    23.943] (II) fglrx(0): Manufacturer: SEC  Model: 4141  Serial#: 0
[    23.943] (II) fglrx(0): Year: 2009  Week: 0
[    23.943] (II) fglrx(0): EDID Version: 1.3
[    23.943] (II) fglrx(0): Digital Display Input
[    23.943] (II) fglrx(0): Max Image Size [cm]: horiz.: 34  vert.: 19
[    23.943] (II) fglrx(0): Gamma: 2.20
[    23.943] (II) fglrx(0): No DPMS capabilities specified
[    23.943] (II) fglrx(0): Supported color encodings: RGB 4:4:4 YCrCb 4:4:4 
[    23.943] (II) fglrx(0): First detailed timing is preferred mode
[    23.943] (II) fglrx(0): redX: 0.615 redY: 0.355   greenX: 0.335 greenY: 0.61
0
[    23.943] (II) fglrx(0): blueX: 0.150 blueY: 0.100   whiteX: 0.313 whiteY: 0.
329
[    23.943] (II) fglrx(0): Manufacturer's mask: 0
[    23.943] (II) fglrx(0): Supported detailed timing:
[    23.943] (II) fglrx(0): clock: 72.3 MHz   Image Size:  344 x 193 mm
[    23.943] (II) fglrx(0): h_active: 1366  h_sync: 1414  h_sync_end 1446 h_blan
k_end 1526 h_border: 0
[    23.943] (II) fglrx(0): v_active: 768  v_sync: 770  v_sync_end 775 v_blankin
g: 790 v_border: 0
[    23.944] (II) fglrx(0): Unknown vendor-specific block f
[    23.944] (II) fglrx(0):  SAMSUNG
[    23.944] (II) fglrx(0):  156AT02-A04
[    23.944] (II) fglrx(0): EDID (in hex):
[    23.944] (II) fglrx(0): 	00ffffffffffff004ca3414100000000
[    23.944] (II) fglrx(0): 	00130103802213780a8da59d5b559c26
[    23.944] (II) fglrx(0): 	19505400000001010101010101010101
[    23.944] (II) fglrx(0): 	010101010101411c56a0500016303020
[    23.944] (II) fglrx(0): 	250058c1100000190000000f00000000
[    23.944] (II) fglrx(0): 	00000000001eb4027400000000fe0053
[    23.944] (II) fglrx(0): 	414d53554e470a2020202020000000fe
[    23.944] (II) fglrx(0): 	00313536415430322d4130340a2000ef
[    23.944] (II) fglrx(0): End of Display0 EDID data --------------------
[    24.032] (II) fglrx(0): EDID for output LVDS
[    24.032] (II) fglrx(0): Manufacturer: SEC  Model: 4141  Serial#: 0
[    24.032] (II) fglrx(0): Year: 2009  Week: 0
[    24.032] (II) fglrx(0): EDID Version: 1.3
[    24.032] (II) fglrx(0): Digital Display Input
[    24.032] (II) fglrx(0): Max Image Size [cm]: horiz.: 34  vert.: 19
[    24.032] (II) fglrx(0): Gamma: 2.20
[    24.032] (II) fglrx(0): No DPMS capabilities specified
[    24.032] (II) fglrx(0): Supported color encodings: RGB 4:4:4 YCrCb 4:4:4 
[    24.032] (II) fglrx(0): First detailed timing is preferred mode
[    24.032] (II) fglrx(0): redX: 0.615 redY: 0.355   greenX: 0.335 greenY: 0.61
0
[    24.032] (II) fglrx(0): blueX: 0.150 blueY: 0.100   whiteX: 0.313 whiteY: 0.
329
[    24.032] (II) fglrx(0): Manufacturer's mask: 0
[    24.032] (II) fglrx(0): Supported detailed timing:
[    24.032] (II) fglrx(0): clock: 72.3 MHz   Image Size:  344 x 193 mm
[    24.032] (II) fglrx(0): h_active: 1366  h_sync: 1414  h_sync_end 1446 h_blan
k_end 1526 h_border: 0
[    24.032] (II) fglrx(0): v_active: 768  v_sync: 770  v_sync_end 775 v_blankin
g: 790 v_border: 0
[    24.032] (II) fglrx(0): Unknown vendor-specific block f
[    24.032] (II) fglrx(0):  SAMSUNG
[    24.032] (II) fglrx(0):  156AT02-A04
[    24.032] (II) fglrx(0): EDID (in hex):
[    24.032] (II) fglrx(0): 	00ffffffffffff004ca3414100000000
[    24.032] (II) fglrx(0): 	00130103802213780a8da59d5b559c26
[    24.032] (II) fglrx(0): 	19505400000001010101010101010101
[    24.032] (II) fglrx(0): 	010101010101411c56a0500016303020
[    24.032] (II) fglrx(0): 	250058c1100000190000000f00000000
[    24.032] (II) fglrx(0): 	00000000001eb4027400000000fe0053
[    24.032] (II) fglrx(0): 	414d53554e470a2020202020000000fe
[    24.032] (II) fglrx(0): 	00313536415430322d4130340a2000ef
[    24.032] (II) fglrx(0): EDID vendor "SEC", prod id 16705
[    24.032] (II) fglrx(0): Printing DDC gathered Modelines:
[    24.032] (II) fglrx(0): Modeline "1366x768"x0.0   72.33  1366 1414 1446 1526
  768 770 775 790 -hsync -vsync (47.4 kHz eP)
[    24.032] (II) fglrx(0): Printing probed modes for output LVDS
[    24.032] (II) fglrx(0): Modeline "1366x768"x60.0   72.33  1366 1414 1446 152
6  768 770 775 790 +hsync +vsync (47.4 kHz)
[    24.032] (II) fglrx(0): Modeline "1280x768"x60.0   72.33  1280 1414 1446 152
6  768 770 775 790 +hsync +vsync (47.4 kHz)
[    24.032] (II) fglrx(0): Modeline "1280x720"x60.0   72.33  1280 1414 1446 152
6  720 770 775 790 +hsync +vsync (47.4 kHz)
[    24.032] (II) fglrx(0): Modeline "1024x768"x60.0   72.33  1024 1414 1446 152
6  768 770 775 790 +hsync +vsync (47.4 kHz)
[    24.032] (II) fglrx(0): Modeline "1280x600"x60.0   72.33  1280 1414 1446 152
6  600 770 775 790 +hsync +vsync (47.4 kHz)
[    24.032] (II) fglrx(0): Modeline "1024x600"x60.0   72.33  1024 1414 1446 152
6  600 770 775 790 +hsync +vsync (47.4 kHz)
[    24.032] (II) fglrx(0): Modeline "800x600"x60.0   72.33  800 1414 1446 1526 
 600 770 775 790 +hsync +vsync (47.4 kHz)
[    24.032] (II) fglrx(0): Modeline "800x480"x60.0   72.33  800 1414 1446 1526 
 480 770 775 790 +hsync +vsync (47.4 kHz)
[    24.032] (II) fglrx(0): Modeline "720x480"x60.0   72.33  720 1414 1446 1526 
 480 770 775 790 +hsync +vsync (47.4 kHz)
[    24.033] (II) fglrx(0): Modeline "640x480"x60.0   72.33  640 1414 1446 1526 
 480 770 775 790 +hsync +vsync (47.4 kHz)
[    24.033] (II) fglrx(0): EDID for output DFP1
[    24.033] (II) fglrx(0): EDID for output CRT1
[    24.033] (II) fglrx(0): Output LVDS connected
[    24.033] (II) fglrx(0): Output DFP1 disconnected
[    24.033] (II) fglrx(0): Output CRT1 disconnected
[    24.033] (II) fglrx(0): Using exact sizes for initial modes
[    24.033] (II) fglrx(0): Output LVDS using initial mode 1366x768
[    24.033] (II) fglrx(0): Display dimensions: (340, 190) mm
[    24.033] (II) fglrx(0): DPI set to (102, 102)
[    24.033] (II) fglrx(0): Adapter ATI Mobility Radeon HD 4200 Series has 2 con
figurable heads and 1 displays connected.
[    24.033] (==) fglrx(0):  PseudoColor visuals disabled
[    24.033] (II) Loading sub module "ramdac"
[    24.033] (II) LoadModule: "ramdac"
[    24.033] (II) Module "ramdac" already built-in
[    24.033] (==) fglrx(0): NoDRI = NO
[    24.033] (==) fglrx(0): Capabilities: 0x00000000
[    24.033] (==) fglrx(0): CapabilitiesEx: 0x00000000
[    24.033] (==) fglrx(0): OpenGL ClientDriverName: "fglrx_dri.so"
[    24.033] (==) fglrx(0): UseFastTLS=0
[    24.033] (==) fglrx(0): BlockSignalsOnLock=1
[    24.033] (--) Depth 24 pixmap format is 32 bpp
[    24.033] (EE) fglrx(0): atiddxDriScreenInit failed, GPS not been initialized
[    24.033] (WW) fglrx(0): ****************************************************
*******
[    24.033] (WW) fglrx(0): * DRI initialization failed                         
      *
[    24.033] (WW) fglrx(0): * kernel module (fglrx.ko) may be missing or incompa
tible *
[    24.033] (WW) fglrx(0): * 2D and 3D acceleration disabled                   
      *
[    24.033] (WW) fglrx(0): ****************************************************
*******
[    24.050] (II) fglrx(0): FBADPhys: 0xc0000000 FBMappedSize: 0x10000000
[    24.085] (II) fglrx(0): FBMM initialized for area (0,0)-(1600,8191)
[    24.085] (II) fglrx(0): FBMM auto alloc for area (0,0)-(1600,1600) (front co
lor buffer - assumption)
[    24.085] (II) fglrx(0): Largest offscreen area available: 1600 x 6591
[    24.098] (==) fglrx(0): Backing store disabled
[    24.098] (II) Loading extension FGLRXEXTENSION
[    24.098] (**) fglrx(0): DPMS enabled
[    24.098] (II) fglrx(0): Initialized in-driver Xinerama extension
[    24.098] (WW) fglrx(0): Textured Video not supported without DRI enabled.
[    24.098] (II) LoadModule: "glesx"
[    24.098] (II) Loading /usr/lib/x86_64-linux-gnu/xorg/extra-modules/extra-mod
ules.dpkg-tmp/modules/glesx.so
[    24.408] (II) Module glesx: vendor="X.Org Foundation"
[    24.409] 	compiled for 1.4.99.906, module version = 1.0.0
[    24.409] (II) Loading extension GLESX
[    24.409] (II) fglrx(0): GLESX enableFlags = 512
[    24.419] (II) LoadModule: "amdxmm"
[    24.419] (II) Loading /usr/lib/x86_64-linux-gnu/xorg/extra-modules/extra-mod
ules.dpkg-tmp/modules/amdxmm.so
[    24.438] (II) Module amdxmm: vendor="X.Org Foundation"
[    24.438] 	compiled for 1.4.99.906, module version = 2.0.0
[    24.438] (EE) fglrx(0): XMM failed to open CMMQS connection.(EE) fglrx(0): 
[    24.438] (EE) fglrx(0): XMM failed to initialize
[    24.438] (WW) fglrx(0): No XV video playback available
[    24.438] (II) fglrx(0): Enable composite support successfully
[    24.438] (WW) fglrx(0): Option "VendorName" is not used
[    24.438] (WW) fglrx(0): Option "ModelName" is not used
[    24.438] (==) fglrx(0): Silken mouse enabled
[    24.438] (==) fglrx(0): Using HW cursor of display infrastructure!
[    24.438] (II) fglrx(0): Disabling in-server RandR and enabling in-driver Ran
dR 1.2.
[    24.438] (II) fglrx(0): 'LVDS LCD' ConnectorType, abstracted as 'Panel'
[    24.438] (II) fglrx(0): 'eDP LCD' ConnectorType, abstracted as 'Panel'
[    24.448] (WW) fglrx(0): Unknown ConnectorType: 0!
[    25.056] (--) RandR disabled
[    25.056] (II) Initializing built-in extension Generic Event Extension
[    25.056] (II) Initializing built-in extension SHAPE
[    25.056] (II) Initializing built-in extension MIT-SHM
[    25.056] (II) Initializing built-in extension XInputExtension
[    25.056] (II) Initializing built-in extension XTEST
[    25.056] (II) Initializing built-in extension BIG-REQUESTS
[    25.056] (II) Initializing built-in extension SYNC
[    25.056] (II) Initializing built-in extension XKEYBOARD
[    25.056] (II) Initializing built-in extension XC-MISC
[    25.056] (II) Initializing built-in extension SECURITY
[    25.056] (II) Initializing built-in extension XINERAMA
[    25.056] (II) Initializing built-in extension XFIXES
[    25.056] (II) Initializing built-in extension RENDER
[    25.056] (II) Initializing built-in extension RANDR
[    25.056] (II) Initializing built-in extension COMPOSITE
[    25.056] (II) Initializing built-in extension DAMAGE
[    25.059] (II) AIGLX: Screen 0 is not DRI capable
[    25.060] [glesx] __glESXExtensionInit: No GL ES2.0 capable screen found!
[    25.081] (II) fglrx(0): Setting screen physical size to 361 x 203
[    25.198] (II) XKB: reuse xkmfile /var/lib/xkb/server-B20D7FC79C7F597315E3E50
1AEF10E0D866E8E92.xkm
[    25.209] (II) config/udev: Adding input device Power Button (/dev/input/even
t3)
[    25.209] (**) Power Button: Applying InputClass "evdev keyboard catchall"
[    25.209] (II) LoadModule: "evdev"
[    25.209] (II) Loading /usr/lib/xorg/modules/input/evdev_drv.so
[    25.226] (II) Module evdev: vendor="X.Org Foundation"
[    25.226] 	compiled for 1.12.3, module version = 2.7.0
[    25.226] 	Module class: X.Org XInput Driver
[    25.226] 	ABI class: X.Org XInput driver, version 16.0
[    25.226] (II) Using input driver 'evdev' for 'Power Button'
[    25.226] (**) Power Button: always reports core events
[    25.226] (**) evdev: Power Button: Device: "/dev/input/event3"
[    25.226] (--) evdev: Power Button: Vendor 0 Product 0x1
[    25.226] (--) evdev: Power Button: Found keys
[    25.226] (II) evdev: Power Button: Configuring as keyboard
[    25.226] (**) Option "config_info" "udev:/sys/devices/LNXSYSTM:00/LNXPWRBN:0
0/input/input3/event3"
[    25.226] (II) XINPUT: Adding extended input device "Power Button" (type: KEY
BOARD, id 6)
[    25.226] (**) Option "xkb_rules" "evdev"
[    25.227] (**) Option "xkb_model" "pc105"
[    25.227] (**) Option "xkb_layout" "it"
[    25.229] (II) XKB: reuse xkmfile /var/lib/xkb/server-3781FECB9CB8D26EE03343D
B2C93394EA704B98F.xkm
[    25.242] (II) config/udev: Adding input device Video Bus (/dev/input/event5)
[    25.242] (**) Video Bus: Applying InputClass "evdev keyboard catchall"
[    25.242] (II) Using input driver 'evdev' for 'Video Bus'
[    25.242] (**) Video Bus: always reports core events
[    25.242] (**) evdev: Video Bus: Device: "/dev/input/event5"
[    25.242] (--) evdev: Video Bus: Vendor 0 Product 0x6
[    25.242] (--) evdev: Video Bus: Found keys
[    25.242] (II) evdev: Video Bus: Configuring as keyboard
[    25.242] (**) Option "config_info" "udev:/sys/devices/LNXSYSTM:00/device:00/
PNP0A08:00/device:35/LNXVIDEO:01/input/input5/event5"
[    25.242] (II) XINPUT: Adding extended input device "Video Bus" (type: KEYBOA
RD, id 7)
[    25.242] (**) Option "xkb_rules" "evdev"
[    25.242] (**) Option "xkb_model" "pc105"
[    25.242] (**) Option "xkb_layout" "it"
[    25.243] (II) config/udev: Adding input device Power Button (/dev/input/even
t0)
[    25.243] (**) Power Button: Applying InputClass "evdev keyboard catchall"
[    25.243] (II) Using input driver 'evdev' for 'Power Button'
[    25.243] (**) Power Button: always reports core events
[    25.243] (**) evdev: Power Button: Device: "/dev/input/event0"
[    25.243] (--) evdev: Power Button: Vendor 0 Product 0x1
[    25.243] (--) evdev: Power Button: Found keys
[    25.243] (II) evdev: Power Button: Configuring as keyboard
[    25.243] (**) Option "config_info" "udev:/sys/devices/LNXSYSTM:00/device:00/
PNP0C0C:00/input/input0/event0"
[    25.243] (II) XINPUT: Adding extended input device "Power Button" (type: KEY
BOARD, id 8)
[    25.243] (**) Option "xkb_rules" "evdev"
[    25.243] (**) Option "xkb_model" "pc105"
[    25.243] (**) Option "xkb_layout" "it"
[    25.244] (II) config/udev: Adding input device Lid Switch (/dev/input/event1
)
[    25.244] (II) No input driver specified, ignoring this device.
[    25.244] (II) This device may have been added with another device file.
[    25.244] (II) config/udev: Adding input device Sleep Button (/dev/input/even
t2)
[    25.244] (**) Sleep Button: Applying InputClass "evdev keyboard catchall"
[    25.244] (II) Using input driver 'evdev' for 'Sleep Button'
[    25.244] (**) Sleep Button: always reports core events
[    25.244] (**) evdev: Sleep Button: Device: "/dev/input/event2"
[    25.244] (--) evdev: Sleep Button: Vendor 0 Product 0x3
[    25.244] (--) evdev: Sleep Button: Found keys
[    25.244] (II) evdev: Sleep Button: Configuring as keyboard
[    25.244] (**) Option "config_info" "udev:/sys/devices/LNXSYSTM:00/device:00/
PNP0C0E:00/input/input2/event2"
[    25.244] (II) XINPUT: Adding extended input device "Sleep Button" (type: KEY
BOARD, id 9)
[    25.244] (**) Option "xkb_rules" "evdev"
[    25.244] (**) Option "xkb_model" "pc105"
[    25.244] (**) Option "xkb_layout" "it"
[    25.245] (II) config/udev: Adding input device CNF7017 (/dev/input/event8)
[    25.245] (**) CNF7017: Applying InputClass "evdev keyboard catchall"
[    25.245] (II) Using input driver 'evdev' for 'CNF7017'
[    25.245] (**) CNF7017: always reports core events
[    25.245] (**) evdev: CNF7017: Device: "/dev/input/event8"
[    25.245] (--) evdev: CNF7017: Vendor 0x4f2 Product 0xb044
[    25.245] (--) evdev: CNF7017: Found keys
[    25.245] (II) evdev: CNF7017: Configuring as keyboard
[    25.245] (**) Option "config_info" "udev:/sys/devices/pci0000:00/0000:00:13.
2/usb2/2-5/2-5:1.0/input/input8/event8"
[    25.245] (II) XINPUT: Adding extended input device "CNF7017" (type: KEYBOARD
, id 10)
[    25.245] (**) Option "xkb_rules" "evdev"
[    25.245] (**) Option "xkb_model" "pc105"
[    25.245] (**) Option "xkb_layout" "it"
[    25.245] (II) config/udev: Adding input device HDA ATI SB Mic (/dev/input/ev
ent10)
[    25.245] (II) No input driver specified, ignoring this device.
[    25.245] (II) This device may have been added with another device file.
[    25.246] (II) config/udev: Adding input device HDA ATI SB Headphone (/dev/in
put/event11)
[    25.246] (II) No input driver specified, ignoring this device.
[    25.246] (II) This device may have been added with another device file.
[    25.246] (II) config/udev: Adding input device HDA ATI SB Line (/dev/input/e
vent9)
[    25.246] (II) No input driver specified, ignoring this device.
[    25.246] (II) This device may have been added with another device file.
[    25.246] (II) config/udev: Adding input device AT Translated Set 2 keyboard 
(/dev/input/event4)
[    25.246] (**) AT Translated Set 2 keyboard: Applying InputClass "evdev keybo
ard catchall"
[    25.246] (II) Using input driver 'evdev' for 'AT Translated Set 2 keyboard'
[    25.246] (**) AT Translated Set 2 keyboard: always reports core events
[    25.246] (**) evdev: AT Translated Set 2 keyboard: Device: "/dev/input/event
4"
[    25.246] (--) evdev: AT Translated Set 2 keyboard: Vendor 0x1 Product 0x1
[    25.246] (--) evdev: AT Translated Set 2 keyboard: Found keys
[    25.246] (II) evdev: AT Translated Set 2 keyboard: Configuring as keyboard
[    25.246] (**) Option "config_info" "udev:/sys/devices/platform/i8042/serio0/
input/input4/event4"
[    25.246] (II) XINPUT: Adding extended input device "AT Translated Set 2 keyb
oard" (type: KEYBOARD, id 11)
[    25.246] (**) Option "xkb_rules" "evdev"
[    25.246] (**) Option "xkb_model" "pc105"
[    25.246] (**) Option "xkb_layout" "it"
[    25.247] (II) config/udev: Adding input device SynPS/2 Synaptics TouchPad (/
dev/input/event7)
[    25.247] (**) SynPS/2 Synaptics TouchPad: Applying InputClass "evdev touchpa
d catchall"
[    25.247] (**) SynPS/2 Synaptics TouchPad: Applying InputClass "touchpad catc
hall"
[    25.247] (**) SynPS/2 Synaptics TouchPad: Applying InputClass "Default click
pad buttons"
[    25.247] (II) LoadModule: "synaptics"
[    25.247] (II) Loading /usr/lib/xorg/modules/input/synaptics_drv.so
[    25.257] (II) Module synaptics: vendor="X.Org Foundation"
[    25.257] 	compiled for 1.12.3, module version = 1.6.99
[    25.257] 	Module class: X.Org XInput Driver
[    25.257] 	ABI class: X.Org XInput driver, version 16.0
[    25.257] (II) Using input driver 'synaptics' for 'SynPS/2 Synaptics TouchPad
'
[    25.257] (**) SynPS/2 Synaptics TouchPad: always reports core events
[    25.257] (**) Option "Device" "/dev/input/event7"
[    25.257] (II) synaptics: SynPS/2 Synaptics TouchPad: ignoring touch events f
or semi-multitouch device
[    25.257] (--) synaptics: SynPS/2 Synaptics TouchPad: x-axis range 1472 - 572
0
[    25.257] (--) synaptics: SynPS/2 Synaptics TouchPad: y-axis range 1408 - 478
0
[    25.257] (--) synaptics: SynPS/2 Synaptics TouchPad: pressure range 0 - 255
[    25.257] (--) synaptics: SynPS/2 Synaptics TouchPad: finger width range 0 - 
15
[    25.257] (--) synaptics: SynPS/2 Synaptics TouchPad: buttons: left right dou
ble triple scroll-buttons
[    25.257] (--) synaptics: SynPS/2 Synaptics TouchPad: Vendor 0x2 Product 0x7
[    25.257] (--) synaptics: SynPS/2 Synaptics TouchPad: touchpad found
[    25.257] (**) SynPS/2 Synaptics TouchPad: always reports core events
[    25.258] (**) Option "config_info" "udev:/sys/devices/platform/i8042/serio2/
input/input7/event7"
[    25.258] (II) XINPUT: Adding extended input device "SynPS/2 Synaptics TouchP
ad" (type: TOUCHPAD, id 12)
[    25.258] (**) synaptics: SynPS/2 Synaptics TouchPad: (accel) MinSpeed is now
 constant deceleration 2.5
[    25.258] (**) synaptics: SynPS/2 Synaptics TouchPad: (accel) MaxSpeed is now
 1.75
[    25.258] (**) synaptics: SynPS/2 Synaptics TouchPad: (accel) AccelFactor is 
now 0.037
[    25.258] (**) SynPS/2 Synaptics TouchPad: (accel) keeping acceleration schem
e 1
[    25.258] (**) SynPS/2 Synaptics TouchPad: (accel) acceleration profile 1
[    25.258] (**) SynPS/2 Synaptics TouchPad: (accel) acceleration factor: 2.000
[    25.258] (**) SynPS/2 Synaptics TouchPad: (accel) acceleration threshold: 4
[    25.258] (--) synaptics: SynPS/2 Synaptics TouchPad: touchpad found
[    25.258] (II) config/udev: Adding input device SynPS/2 Synaptics TouchPad (/
dev/input/mouse0)
[    25.258] (**) SynPS/2 Synaptics TouchPad: Ignoring device from InputClass "t
ouchpad ignore duplicates"
[    25.260] (II) config/udev: Adding input device Acer BMA150 accelerometer (/d
ev/input/event6)
[    25.260] (II) No input driver specified, ignoring this device.
[    25.260] (II) This device may have been added with another device file.
[    25.261] (II) config/udev: Adding input device Acer BMA150 accelerometer (/d
ev/input/js0)
[    25.261] (II) No input driver specified, ignoring this device.
[    25.261] (II) This device may have been added with another device file.
[    25.440] (II) fglrx(0): Restoring Recent Mode via PCS is not supported in RA
NDR 1.2 capable environments
[    29.936] (II) fglrx(0): EDID vendor "SEC", prod id 16705
[    29.936] (II) fglrx(0): Printing DDC gathered Modelines:
[    29.936] (II) fglrx(0): Modeline "1366x768"x0.0   72.33  1366 1414 1446 1526
  768 770 775 790 -hsync -vsync (47.4 kHz eP)
[    34.627] (II) fglrx(0): EDID vendor "SEC", prod id 16705
[    34.627] (II) fglrx(0): Printing DDC gathered Modelines:
[    34.627] (II) fglrx(0): Modeline "1366x768"x0.0   72.33  1366 1414 1446 1526
  768 770 775 790 -hsync -vsync (47.4 kHz eP)
[    40.106] (II) fglrx(0): EDID vendor "SEC", prod id 16705
[    40.106] (II) fglrx(0): Printing DDC gathered Modelines:
[    40.106] (II) fglrx(0): Modeline "1366x768"x0.0   72.33  1366 1414 1446 1526
  768 770 775 790 -hsync -vsync (47.4 kHz eP)
[    40.474] (II) fglrx(0): EDID vendor "SEC", prod id 16705
[    40.474] (II) fglrx(0): Printing DDC gathered Modelines:
[    40.474] (II) fglrx(0): Modeline "1366x768"x0.0   72.33  1366 1414 1446 1526
  768 770 775 790 -hsync -vsync (47.4 kHz eP)
[    41.916] (II) fglrx(0): EDID vendor "SEC", prod id 16705
[    41.917] (II) fglrx(0): Printing DDC gathered Modelines:
[    41.917] (II) fglrx(0): Modeline "1366x768"x0.0   72.33  1366 1414 1446 1526
  768 770 775 790 -hsync -vsync (47.4 kHz eP)
[   114.073] (II) fglrx(0): EDID vendor "SEC", prod id 16705
[   114.074] (II) fglrx(0): Printing DDC gathered Modelines:
[   114.074] (II) fglrx(0): Modeline "1366x768"x0.0   72.33  1366 1414 1446 1526
  768 770 775 790 -hsync -vsync (47.4 kHz eP)










.

DarkNeo
Linux 4.x
Linux 4.x
Messaggi: 1071
Iscritto il: sab 4 giu 2005, 0:00
Nome Cognome: Stefano Arbitrio
Slackware: Slackware64 14
Kernel: 3.2.X
Desktop: KDE
Località: Matrix

Re: Ubuntu 12.10 Vs Ati catalist 12.6

Messaggio da DarkNeo »

questo è xorg.conf

Codice: Seleziona tutto

Section "ServerLayout"
	Identifier     "aticonfig Layout"
	Screen      0  "aticonfig-Screen[0]-0" 0 0
EndSection

Section "Module"
EndSection

Section "Monitor"
	Identifier   "aticonfig-Monitor[0]-0"
	Option	    "VendorName" "ATI Proprietary Driver"
	Option	    "ModelName" "Generic Autodetecting Monitor"
	Option	    "DPMS" "true"
EndSection

Section "Device"
	Identifier  "aticonfig-Device[0]-0"
	Driver      "fglrx"
	BusID       "PCI:1:5:0"
EndSection

Section "Screen"
	Identifier "aticonfig-Screen[0]-0"
	Device     "aticonfig-Device[0]-0"
	Monitor    "aticonfig-Monitor[0]-0"
	DefaultDepth     24
	SubSection "Display"
		Viewport   0 0
		Depth     24
	EndSubSection
EndSection
La guida che ho seguito è sempre la stessa
http://www.chimerarevo.com/ubuntu-12-10 ... -problema/

Avatar utente
414N
Iper Master
Iper Master
Messaggi: 2922
Iscritto il: mer 13 feb 2008, 16:19
Slackware: 15.0
Kernel: 5.15.19
Desktop: KDE5
Località: Bulagna
Contatta:

Re: Ubuntu 12.10 Vs Ati catalist 12.6

Messaggio da 414N »

DarkNeo ha scritto:Ah, non lo sapevo, comunque l'errore che ho nella partizione ubuntu nel /var/log/Xorg.0.log è questo:

Codice: Seleziona tutto

    24.033] (EE) fglrx(0): atiddxDriScreenInit failed, GPS not been initialized
[    24.033] (WW) fglrx(0): ****************************************************
*******
[    24.033] (WW) fglrx(0): * DRI initialization failed                         
      *
[    24.033] (WW) fglrx(0): * kernel module (fglrx.ko) may be missing or incompa
tible *
[    24.033] (WW) fglrx(0): * 2D and 3D acceleration disabled                   
      *
Il modulo fglrx è stato caricato al boot oppure prima che parta X?

DarkNeo
Linux 4.x
Linux 4.x
Messaggi: 1071
Iscritto il: sab 4 giu 2005, 0:00
Nome Cognome: Stefano Arbitrio
Slackware: Slackware64 14
Kernel: 3.2.X
Desktop: KDE
Località: Matrix

Re: Ubuntu 12.10 Vs Ati catalist 12.6

Messaggio da DarkNeo »

Si vede da qui?

Codice: Seleziona tutto

root@DarkNeo:~# dmesg |more
[    0.000000] Initializing cgroup subsys cpuset
[    0.000000] Initializing cgroup subsys cpu
[    0.000000] Linux version 3.5.0-17-generic (buildd@allspice) (gcc version 4.7
.2 (Ubuntu/Linaro 4.7.2-2ubuntu1) ) #28-Ubuntu SMP Tue Oct 9 19:31:23 UTC 2012 (
Ubuntu 3.5.0-17.28-generic 3.5.5)
[    0.000000] Command line: BOOT_IMAGE=/boot/vmlinuz-3.5.0-17-generic root=UUID
=b6fdf641-f65d-4b5c-88fd-14cf062f7785 ro quiet splash vt.handoff=7
[    0.000000] KERNEL supported cpus:
[    0.000000]   Intel GenuineIntel
[    0.000000]   AMD AuthenticAMD
[    0.000000]   Centaur CentaurHauls
[    0.000000] e820: BIOS-provided physical RAM map:
[    0.000000] BIOS-e820: [mem 0x0000000000000000-0x000000000009dbff] usable
[    0.000000] BIOS-e820: [mem 0x000000000009dc00-0x000000000009ffff] reserved
[    0.000000] BIOS-e820: [mem 0x00000000000d0000-0x00000000000fffff] reserved
[    0.000000] BIOS-e820: [mem 0x0000000000100000-0x00000000b7ebffff] usable
[    0.000000] BIOS-e820: [mem 0x00000000b7ec0000-0x00000000b7ed7fff] ACPI data
[    0.000000] BIOS-e820: [mem 0x00000000b7ed8000-0x00000000b7ed9fff] ACPI NVS
[    0.000000] BIOS-e820: [mem 0x00000000b7eda000-0x00000000c7ffffff] reserved
[    0.000000] BIOS-e820: [mem 0x00000000e0000000-0x00000000efffffff] reserved
[    0.000000] BIOS-e820: [mem 0x00000000fec00000-0x00000000fec0ffff] reserved
[    0.000000] BIOS-e820: [mem 0x00000000fee00000-0x00000000fee00fff] reserved
[    0.000000] BIOS-e820: [mem 0x00000000fff00000-0x00000000ffffffff] reserved
[    0.000000] BIOS-e820: [mem 0x0000000100000000-0x0000000137ffffff] usable
[    0.000000] NX (Execute Disable) protection: active
[    0.000000] DMI present.
[    0.000000] DMI: Acer            Aspire 5542                    /JV50TR      
                   , BIOS V1.05           10/22/2009
[    0.000000] e820: update [mem 0x00000000-0x0000ffff] usable ==> reserved
[    0.000000] e820: remove [mem 0x000a0000-0x000fffff] usable
[    0.000000] No AGP bridge found
[    0.000000] e820: last_pfn = 0x138000 max_arch_pfn = 0x400000000
[    0.000000] MTRR default type: uncachable
[    0.000000] MTRR fixed ranges enabled:
[    0.000000]   00000-9FFFF write-back
[    0.000000]   A0000-BFFFF uncachable
[    0.000000]   C0000-D3FFF write-protect
[    0.000000]   D4000-DFFFF uncachable
[    0.000000]   E0000-FFFFF write-protect
[    0.000000] MTRR variable ranges enabled:
[    0.000000]   0 base 000000000000 mask FFFF80000000 write-back
[    0.000000]   1 base 000080000000 mask FFFFE0000000 write-back
[    0.000000]   2 base 0000A0000000 mask FFFFF0000000 write-back
[    0.000000]   3 base 0000B0000000 mask FFFFF8000000 write-back
[    0.000000]   4 base 0000B8000000 mask FFFFFC000000 write-back
[    0.000000]   5 base 0000BC000000 mask FFFFFE000000 write-back
[    0.000000]   6 disabled
[    0.000000]   7 disabled
[    0.000000] TOM2: 0000000138000000 aka 4992M
[    0.000000] x86 PAT enabled: cpu 0, old 0x7040600070406, new 0x7010600070106
[    0.000000] e820: update [mem 0xbe000000-0xffffffff] usable ==> reserved
[    0.000000] e820: last_pfn = 0xb7ec0 max_arch_pfn = 0x400000000
[    0.000000] found SMP MP-table at [mem 0x000f8170-0x000f817f] mapped at [ffff
8800000f8170]
[    0.000000] initial memory mapped: [mem 0x00000000-0x1fffffff]
[    0.000000] Base memory trampoline at [ffff880000097000] 97000 size 24576
[    0.000000] Using GB pages for direct mapping
[    0.000000] init_memory_mapping: [mem 0x00000000-0xb7ebffff]
[    0.000000]  [mem 0x00000000-0x7fffffff] page 1G
[    0.000000]  [mem 0x80000000-0xb7dfffff] page 2M
[    0.000000]  [mem 0xb7e00000-0xb7ebffff] page 4k
[    0.000000] kernel direct mapping tables up to 0xb7ebffff @ [mem 0x1fffd000-0
x1fffffff]
[    0.000000] init_memory_mapping: [mem 0x100000000-0x137ffffff]
[    0.000000]  [mem 0x100000000-0x137ffffff] page 2M
[    0.000000] kernel direct mapping tables up to 0x137ffffff @ [mem 0xb7ebe000-
0xb7ebffff]
[    0.000000] RAMDISK: [mem 0x362e6000-0x3716afff]
[    0.000000] ACPI: RSDP 00000000000f8030 00024 (v02 PTLTD )
[    0.000000] ACPI: XSDT 00000000b7ecb3e6 00054 (v01 ACRSYS ACRPRDCT 06040000 I
NNA 00000000)
[    0.000000] ACPI: FACP 00000000b7ed78ca 000F4 (v03 AMD    ANT      06040000 A
TI  000F4240)
[    0.000000] ACPI: DSDT 00000000b7ecb43a 0C490 (v01    ATI    SB700 06040000 M
SFT 03000001)
[    0.000000] ACPI: FACS 00000000b7ed9fc0 00040
[    0.000000] ACPI: SLIC 00000000b7ed7a32 00176 (v01 ACRSYS ACRPRDCT 06040000 A
NNI 00000001)
[    0.000000] ACPI: SSDT 00000000b7ed7ba8 00386 (v01 AMD    POWERNOW 06040000 A
MD  00000001)
[    0.000000] ACPI: APIC 00000000b7ed7f2e 0005E (v01 PTLTD  ? APIC   06040000  
LTP 00000000)
[    0.000000] ACPI: MCFG 00000000b7ed7f8c 0003C (v01 PTLTD    MCFG   06040000  
LTP 00000000)
[    0.000000] ACPI: HPET 00000000b7ed7fc8 00038 (v01 PTLTD  HPETTBL  06040000  
LTP 00000001)
[    0.000000] ACPI: Local APIC address 0xfee00000
[    0.000000] Scanning NUMA topology in Northbridge 24
[    0.000000] No NUMA configuration found
[    0.000000] Faking a node at [mem 0x0000000000000000-0x0000000137ffffff]
[    0.000000] Initmem setup node 0 [mem 0x00000000-0x137ffffff]
[    0.000000]   NODE_DATA [mem 0x137ffc000-0x137ffffff]
[    0.000000]  [ffffea0000000000-ffffea0004dfffff] PMD -> [ffff880133a00000-fff
f8801375fffff] on node 0
[    0.000000] Zone ranges:
[    0.000000]   DMA      [mem 0x00010000-0x00ffffff]
[    0.000000]   DMA32    [mem 0x01000000-0xffffffff]
[    0.000000]   Normal   [mem 0x100000000-0x137ffffff]
[    0.000000] Movable zone start for each node
[    0.000000] Early memory node ranges
[    0.000000]   node   0: [mem 0x00010000-0x0009cfff]
[    0.000000]   node   0: [mem 0x00100000-0xb7ebffff]
[    0.000000]   node   0: [mem 0x100000000-0x137ffffff]
[    0.000000] On node 0 totalpages: 982605
[    0.000000]   DMA zone: 64 pages used for memmap
[    0.000000]   DMA zone: 6 pages reserved
[    0.000000]   DMA zone: 3911 pages, LIFO batch:0
[    0.000000]   DMA32 zone: 16320 pages used for memmap
[    0.000000]   DMA32 zone: 732928 pages, LIFO batch:31
[    0.000000]   Normal zone: 3584 pages used for memmap
[    0.000000]   Normal zone: 225792 pages, LIFO batch:31
[    0.000000] ACPI: PM-Timer IO Port: 0x8008
[    0.000000] ACPI: Local APIC address 0xfee00000
[    0.000000] ACPI: LAPIC (acpi_id[0x00] lapic_id[0x00] enabled)
[    0.000000] ACPI: LAPIC (acpi_id[0x01] lapic_id[0x01] enabled)
[    0.000000] ACPI: LAPIC_NMI (acpi_id[0x00] high edge lint[0x1])
[    0.000000] ACPI: LAPIC_NMI (acpi_id[0x01] high edge lint[0x1])
[    0.000000] ACPI: IOAPIC (id[0x02] address[0xfec00000] gsi_base[0])
[    0.000000] IOAPIC[0]: apic_id 2, version 33, address 0xfec00000, GSI 0-23
[    0.000000] ACPI: INT_SRC_OVR (bus 0 bus_irq 0 global_irq 2 low level)
[    0.000000] ACPI: IRQ0 used by override.
[    0.000000] ACPI: IRQ2 used by override.
[    0.000000] ACPI: IRQ9 used by override.
[    0.000000] Using ACPI (MADT) for SMP configuration information
[    0.000000] ACPI: HPET id: 0x43538301 base: 0xfed00000
[    0.000000] SMP: Allowing 2 CPUs, 0 hotplug CPUs
[    0.000000] nr_irqs_gsi: 40
[    0.000000] PM: Registered nosave memory: 000000000009d000 - 000000000009e000
[    0.000000] PM: Registered nosave memory: 000000000009e000 - 00000000000a0000
[    0.000000] PM: Registered nosave memory: 00000000000a0000 - 00000000000d0000
[    0.000000] PM: Registered nosave memory: 00000000000d0000 - 0000000000100000
[    0.000000] PM: Registered nosave memory: 00000000b7ec0000 - 00000000b7ed8000
[    0.000000] PM: Registered nosave memory: 00000000b7ed8000 - 00000000b7eda000
[    0.000000] PM: Registered nosave memory: 00000000b7eda000 - 00000000c8000000
[    0.000000] PM: Registered nosave memory: 00000000c8000000 - 00000000e0000000
[    0.000000] PM: Registered nosave memory: 00000000e0000000 - 00000000f0000000
[    0.000000] PM: Registered nosave memory: 00000000f0000000 - 00000000fec00000
[    0.000000] PM: Registered nosave memory: 00000000fec00000 - 00000000fec10000
[    0.000000] PM: Registered nosave memory: 00000000fec10000 - 00000000fee00000
[    0.000000] PM: Registered nosave memory: 00000000fee00000 - 00000000fee01000
[    0.000000] PM: Registered nosave memory: 00000000fee01000 - 00000000fff00000
[    0.000000] PM: Registered nosave memory: 00000000fff00000 - 0000000100000000
[    0.000000] e820: [mem 0xc8000000-0xdfffffff] available for PCI devices
[    0.000000] Booting paravirtualized kernel on bare hardware
[    0.000000] setup_percpu: NR_CPUS:256 nr_cpumask_bits:256 nr_cpu_ids:2 nr_nod
e_ids:1
[    0.000000] PERCPU: Embedded 28 pages/cpu @ffff880137c00000 s83584 r8192 d229
12 u1048576
[    0.000000] pcpu-alloc: s83584 r8192 d22912 u1048576 alloc=1*2097152
[    0.000000] pcpu-alloc: [0] 0 1 
[    0.000000] Built 1 zonelists in Node order, mobility grouping on.  Total pag
es: 962631
[    0.000000] Policy zone: Normal
[    0.000000] Kernel command line: BOOT_IMAGE=/boot/vmlinuz-3.5.0-17-generic ro
ot=UUID=b6fdf641-f65d-4b5c-88fd-14cf062f7785 ro quiet splash vt.handoff=7
[    0.000000] PID hash table entries: 4096 (order: 3, 32768 bytes)
[    0.000000] __ex_table already sorted, skipping sort
[    0.000000] Checking aperture...
[    0.000000] No AGP bridge found
[    0.000000] Node 0: aperture @ 2dfc000000 size 32 MB
[    0.000000] Aperture beyond 4GB. Ignoring.
[    0.000000] Your BIOS doesn't leave a aperture memory hole
[    0.000000] Please enable the IOMMU option in the BIOS setup
[    0.000000] This costs you 64 MB of RAM
[    0.000000] Mapping aperture over 65536 KB of RAM @ ac000000
[    0.000000] PM: Registered nosave memory: 00000000ac000000 - 00000000b0000000
[    0.000000] Memory: 3706452k/5111808k available (6711k kernel code, 1181388k 
absent, 223968k reserved, 6459k data, 928k init)
[    0.000000] SLUB: Genslabs=15, HWalign=64, Order=0-3, MinObjects=0, CPUs=2, N
odes=1
[    0.000000] Hierarchical RCU implementation.
[    0.000000] 	RCU dyntick-idle grace-period acceleration is enabled.
[    0.000000] NR_IRQS:16640 nr_irqs:512 16
[    0.000000] spurious 8259A interrupt: IRQ7.
[    0.000000] vt handoff: transparent VT on vt#7
[    0.000000] Console: colour dummy device 80x25
[    0.000000] console [tty0] enabled
[    0.000000] allocated 15728640 bytes of page_cgroup
[    0.000000] please try 'cgroup_disable=memory' option if you don't want memor
y cgroups
[    0.000000] hpet clockevent registered
[    0.004000] Fast TSC calibration using PIT
[    0.008000] Detected 1994.877 MHz processor.
[    0.000003] Calibrating delay loop (skipped), value calculated using timer fr
equency.. 3989.75 BogoMIPS (lpj=7979508)
[    0.000008] pid_max: default: 32768 minimum: 301
[    0.000036] Security Framework initialized
[    0.000052] AppArmor: AppArmor initialized
[    0.000053] Yama: becoming mindful.
[    0.000418] Dentry cache hash table entries: 524288 (order: 10, 4194304 bytes
)
[    0.001978] Inode-cache hash table entries: 262144 (order: 9, 2097152 bytes)
[    0.002735] Mount-cache hash table entries: 256
[    0.002968] Initializing cgroup subsys cpuacct
[    0.002972] Initializing cgroup subsys memory
[    0.002982] Initializing cgroup subsys devices
[    0.002984] Initializing cgroup subsys freezer
[    0.002985] Initializing cgroup subsys blkio
[    0.002987] Initializing cgroup subsys perf_event
[    0.003016] tseg: 00b7f00000
[    0.003018] CPU: Physical Processor ID: 0
[    0.003019] CPU: Processor Core ID: 0
[    0.003021] mce: CPU supports 6 MCE banks
[    0.003028] LVT offset 0 assigned for vector 0xf9
[    0.003033] using AMD E400 aware idle routine
[    0.004812] ACPI: Core revision 20120320
[    0.823980] ftrace: allocating 25118 entries in 99 pages
[    0.837998] ..TIMER: vector=0x30 apic1=0 pin1=2 apic2=-1 pin2=-1
[    0.877662] CPU0: AMD Athlon(tm) II Dual-Core M300 stepping 02
[    0.981773] Performance Events: AMD PMU driver.
[    0.981778] ... version:                0
[    0.981779] ... bit width:              48
[    0.981781] ... generic registers:      4
[    0.981782] ... value mask:             0000ffffffffffff
[    0.981783] ... max period:             00007fffffffffff
[    0.981784] ... fixed-purpose events:   0
[    0.981785] ... event mask:             000000000000000f
[    0.981954] NMI watchdog: enabled on all CPUs, permanently consumes one hw-PM
U counter.
[    0.982049] Booting Node   0, Processors  #1 Ok.
[    0.995174] Brought up 2 CPUs
[    0.995177] Total of 2 processors activated (7979.50 BogoMIPS).
[    0.995185] System has AMD C1E enabled
[    0.995223] Switch to broadcast mode on CPU1
[    0.995689] Switch to broadcast mode on CPU0
[    0.995961] devtmpfs: initialized
[    0.997519] EVM: security.selinux
[    0.997521] EVM: security.SMACK64
[    0.997522] EVM: security.capability
[    0.997632] PM: Registering ACPI NVS region [mem 0xb7ed8000-0xb7ed9fff] (8192
 bytes)
[    0.998529] dummy: 
[    0.998593] RTC time:  1:32:42, date: 01/14/13
[    0.998635] NET: Registered protocol family 16
[    0.998781] Trying to unpack rootfs image as initramfs...
[    0.998783] node 0 link 0: io port [1000, ffff]
[    0.998788] TOM: 00000000c8000000 aka 3200M
[    0.998790] Fam 10h mmconf [mem 0xe0000000-0xefffffff]
[    0.998793] node 0 link 0: mmio [a0000, bffff]
[    0.998797] node 0 link 0: mmio [c8000000, cfcfffff]
[    0.998800] node 0 link 0: mmio [cfd00000, cfefffff]
[    0.998802] node 0 link 0: mmio [cff00000, cfffffff]
[    0.998804] node 0 link 0: mmio [d0000000, efffffff] ==> [d0000000, dfffffff]
[    0.998807] node 0 link 0: mmio [f0000000, ffffffff]
[    0.998809] TOM2: 0000000138000000 aka 4992M
[    0.998811] bus: [00, ff] on node 0 link 0
[    0.998813] bus: 00 [io  0x0000-0xffff]
[    0.998815] bus: 00 [mem 0x000a0000-0x000bffff]
[    0.998816] bus: 00 [mem 0xc8000000-0xdfffffff]
[    0.998818] bus: 00 [mem 0xf0000000-0xffffffff]
[    0.998819] bus: 00 [mem 0x138000000-0xfcffffffff]
[    0.998938] ACPI FADT declares the system doesn't support PCIe ASPM, so disab
le it
[    0.998940] ACPI: bus type pci registered
[    0.999037] PCI: MMCONFIG for domain 0000 [bus 00-09] at [mem 0xe0000000-0xe0
9fffff] (base 0xe0000000)
[    0.999040] PCI: MMCONFIG at [mem 0xe0000000-0xe09fffff] reserved in E820
[    0.999646] PCI: Using configuration type 1 for base access
[    0.999810] mtrr: your CPUs had inconsistent variable MTRR settings
[    0.999811] mtrr: probably your BIOS does not setup all CPUs.
[    0.999812] mtrr: corrected configuration.
[    1.000710] bio: create slab <bio-0> at 0
[    1.000817] ACPI: Added _OSI(Module Device)
[    1.000819] ACPI: Added _OSI(Processor Device)
[    1.000821] ACPI: Added _OSI(3.0 _SCP Extensions)
[    1.000822] ACPI: Added _OSI(Processor Aggregator Device)
[    1.002180] ACPI: EC: Look up EC in DSDT
[    1.005717] [Firmware Bug]: ACPI: BIOS _OSI(Linux) query ignored
[    1.006256] ACPI: OEMN 00000000b7ed8c69 00173 (v01 AMD    NAHP     00000001 M
SFT 03000001)
[    1.006611] ACPI: Dynamic OEM Table Load:
[    1.006614] ACPI: OEMN           (null) 00173 (v01 AMD    NAHP     00000001 M
SFT 03000001)
[    1.021810] ACPI: Interpreter enabled
[    1.021819] ACPI: (supports S0 S3 S4 S5)
[    1.021843] ACPI: Using IOAPIC for interrupt routing
[    1.034213] ACPI: EC: GPE = 0x3, I/O: command/status = 0x66, data = 0x62
[    1.034424] ACPI: No dock devices found.
[    1.034429] PCI: Using host bridge windows from ACPI; if necessary, use "pci=
nocrs" and report a bug
[    1.035310] ACPI: PCI Root Bridge [PCI0] (domain 0000 [bus 00-ff])
[    1.036586] pci_root PNP0A08:00: >host bridge window [mem 0x000a0000-0x000bff
ff]
[    1.036590] pci_root PNP0A08:00: >host bridge window [mem 0x000d4000-0x000d5f
ff]
[    1.036593] pci_root PNP0A08:00: >host bridge window [mem 0x000d6000-0x000d7f
ff]
[    1.036595] pci_root PNP0A08:00: >host bridge window [mem 0x000d8000-0x000d9f
ff]
[    1.036597] pci_root PNP0A08:00: >host bridge window [mem 0x000da000-0x000dbf
ff]
[    1.036599] pci_root PNP0A08:00: >host bridge window [mem 0x000dc000-0x000ddf
ff]
[    1.036601] pci_root PNP0A08:00: >host bridge window [mem 0x000de000-0x000dff
ff]
[    1.036604] pci_root PNP0A08:00: >host bridge window [mem 0xc8000000-0xdfffff
ff]
[    1.036606] pci_root PNP0A08:00: >host bridge window [mem 0xf0000000-0xffffff
ff]
[    1.036609] pci_root PNP0A08:00: >host bridge window [io  0x0000-0x0cf7]
[    1.036611] pci_root PNP0A08:00: >host bridge window [io  0x0d00-0xffff]
[    1.036650] PCI host bridge to bus 0000:00
[    1.036653] pci_bus 0000:00: >root bus resource [mem 0x000a0000-0x000bffff]
[    1.036655] pci_bus 0000:00: >root bus resource [mem 0x000d4000-0x000d5fff]
[    1.036658] pci_bus 0000:00: >root bus resource [mem 0x000d6000-0x000d7fff]
[    1.036660] pci_bus 0000:00: >root bus resource [mem 0x000d8000-0x000d9fff]
[    1.036662] pci_bus 0000:00: >root bus resource [mem 0x000da000-0x000dbfff]
[    1.036664] pci_bus 0000:00: >root bus resource [mem 0x000dc000-0x000ddfff]
[    1.036666] pci_bus 0000:00: >root bus resource [mem 0x000de000-0x000dffff]
[    1.036668] pci_bus 0000:00: >root bus resource [mem 0xc8000000-0xdfffffff]
[    1.036671] pci_bus 0000:00: >root bus resource [mem 0xf0000000-0xffffffff]
[    1.036673] pci_bus 0000:00: >root bus resource [io  0x0000-0x0cf7]
[    1.036675] pci_bus 0000:00: >root bus resource [io  0x0d00-0xffff]
[    1.036689] pci 0000:00:00.0: >[1022:9601] type 00 class 0x060000
[    1.036748] pci 0000:00:01.0: >[1025:9602] type 01 class 0x060400
[    1.036824] pci 0000:00:04.0: >[1022:9604] type 01 class 0x060400
[    1.036864] pci 0000:00:04.0: >PME# supported from D0 D3hot D3cold
[    1.036916] pci 0000:00:06.0: >[1022:9606] type 01 class 0x060400
[    1.036955] pci 0000:00:06.0: >PME# supported from D0 D3hot D3cold
[    1.037023] pci 0000:00:11.0: >[1002:4391] type 00 class 0x010601
[    1.037044] pci 0000:00:11.0: >reg 10: [io  0x8420-0x8427]
[    1.037054] pci 0000:00:11.0: >reg 14: [io  0x8414-0x8417]
[    1.037065] pci 0000:00:11.0: >reg 18: [io  0x8418-0x841f]
[    1.037075] pci 0000:00:11.0: >reg 1c: [io  0x8410-0x8413]
[    1.037085] pci 0000:00:11.0: >reg 20: [io  0x8400-0x840f]
[    1.037096] pci 0000:00:11.0: >reg 24: [mem 0xf0208000-0xf02083ff]
[    1.037158] pci 0000:00:12.0: >[1002:4397] type 00 class 0x0c0310
[    1.037172] pci 0000:00:12.0: >reg 10: [mem 0xf0004000-0xf0004fff]
[    1.037244] pci 0000:00:12.1: >[1002:4398] type 00 class 0x0c0310
[    1.037258] pci 0000:00:12.1: >reg 10: [mem 0xf0005000-0xf0005fff]
[    1.037333] pci 0000:00:12.2: >[1002:4396] type 00 class 0x0c0320
[    1.037353] pci 0000:00:12.2: >reg 10: [mem 0xf0208400-0xf02084ff]
[    1.037441] pci 0000:00:12.2: >supports D1 D2
[    1.037443] pci 0000:00:12.2: >PME# supported from D0 D1 D2 D3hot
[    1.037468] pci 0000:00:13.0: >[1002:4397] type 00 class 0x0c0310
[    1.037482] pci 0000:00:13.0: >reg 10: [mem 0xf0006000-0xf0006fff]
[    1.037552] pci 0000:00:13.1: >[1002:4398] type 00 class 0x0c0310
[    1.037567] pci 0000:00:13.1: >reg 10: [mem 0xf0007000-0xf0007fff]
[    1.037642] pci 0000:00:13.2: >[1002:4396] type 00 class 0x0c0320
[    1.037662] pci 0000:00:13.2: >reg 10: [mem 0xf0208800-0xf02088ff]
[    1.037750] pci 0000:00:13.2: >supports D1 D2
[    1.037752] pci 0000:00:13.2: >PME# supported from D0 D1 D2 D3hot
[    1.037780] pci 0000:00:14.0: >[1002:4385] type 00 class 0x0c0500
[    1.037889] pci 0000:00:14.2: >[1002:4383] type 00 class 0x040300
[    1.037912] pci 0000:00:14.2: >reg 10: [mem 0xf0000000-0xf0003fff 64bit]
[    1.037982] pci 0000:00:14.2: >PME# supported from D0 D3hot D3cold
[    1.037999] pci 0000:00:14.3: >[1002:439d] type 00 class 0x060100
[    1.038082] pci 0000:00:14.4: >[1002:4384] type 01 class 0x060401
[    1.038132] pci 0000:00:18.0: >[1022:1200] type 00 class 0x060000
[    1.038152] pci 0000:00:18.1: >[1022:1201] type 00 class 0x060000
[    1.038167] pci 0000:00:18.2: >[1022:1202] type 00 class 0x060000
[    1.038183] pci 0000:00:18.3: >[1022:1203] type 00 class 0x060000
[    1.038202] pci 0000:00:18.4: >[1022:1204] type 00 class 0x060000
[    1.038284] pci 0000:01:05.0: >[1002:9712] type 00 class 0x030000
[    1.038294] pci 0000:01:05.0: >reg 10: [mem 0xd0000000-0xdfffffff pref]
[    1.038299] pci 0000:01:05.0: >reg 14: [io  0x9000-0x90ff]
[    1.038305] pci 0000:01:05.0: >reg 18: [mem 0xcfdf0000-0xcfdfffff]
[    1.038316] pci 0000:01:05.0: >reg 24: [mem 0xcfe00000-0xcfefffff]
[    1.038335] pci 0000:01:05.0: >supports D1 D2
[    1.038347] pci 0000:01:05.1: >[1002:970f] type 00 class 0x040300
[    1.038356] pci 0000:01:05.1: >reg 10: [mem 0xcfdec000-0xcfdeffff]
[    1.038390] pci 0000:01:05.1: >supports D1 D2
[    1.038464] pci 0000:00:01.0: >PCI bridge to [bus 01-01]
[    1.038468] pci 0000:00:01.0: >  bridge window [io  0x9000-0x9fff]
[    1.038471] pci 0000:00:01.0: >  bridge window [mem 0xcfd00000-0xcfefffff]
[    1.038475] pci 0000:00:01.0: >  bridge window [mem 0xd0000000-0xdfffffff 64b
it pref]
[    1.038523] pci 0000:03:00.0: >[14e4:1698] type 00 class 0x020000
[    1.038542] pci 0000:03:00.0: >reg 10: [mem 0xf0300000-0xf030ffff 64bit]
[    1.038646] pci 0000:03:00.0: >PME# supported from D3hot D3cold
[    1.043296] pci 0000:00:04.0: >PCI bridge to [bus 03-03]
[    1.043307] pci 0000:00:04.0: >  bridge window [mem 0xf0300000-0xf03fffff]
[    1.043358] pci 0000:09:00.0: >[168c:002a] type 00 class 0x028000
[    1.043379] pci 0000:09:00.0: >reg 10: [mem 0xf0400000-0xf040ffff 64bit]
[    1.043470] pci 0000:09:00.0: >supports D1
[    1.043473] pci 0000:09:00.0: >PME# supported from D0 D1 D3hot
[    1.051291] pci 0000:00:06.0: >PCI bridge to [bus 09-09]
[    1.051301] pci 0000:00:06.0: >  bridge window [mem 0xf0400000-0xf04fffff]
[    1.051407] pci 0000:00:14.4: >PCI bridge to [bus 0a-0a] (subtractive decode)
[    1.051417] pci 0000:00:14.4: >  bridge window [mem 0x000a0000-0x000bffff] (s
ubtractive decode)
[    1.051419] pci 0000:00:14.4: >  bridge window [mem 0x000d4000-0x000d5fff] (s
ubtractive decode)
[    1.051421] pci 0000:00:14.4: >  bridge window [mem 0x000d6000-0x000d7fff] (s
ubtractive decode)
[    1.051423] pci 0000:00:14.4: >  bridge window [mem 0x000d8000-0x000d9fff] (s
ubtractive decode)
[    1.051426] pci 0000:00:14.4: >  bridge window [mem 0x000da000-0x000dbfff] (s
ubtractive decode)
[    1.051428] pci 0000:00:14.4: >  bridge window [mem 0x000dc000-0x000ddfff] (s
ubtractive decode)
[    1.051430] pci 0000:00:14.4: >  bridge window [mem 0x000de000-0x000dffff] (s
ubtractive decode)
[    1.051432] pci 0000:00:14.4: >  bridge window [mem 0xc8000000-0xdfffffff] (s
ubtractive decode)
[    1.051434] pci 0000:00:14.4: >  bridge window [mem 0xf0000000-0xffffffff] (s
ubtractive decode)
[    1.051436] pci 0000:00:14.4: >  bridge window [io  0x0000-0x0cf7] (subtracti
ve decode)
[    1.051438] pci 0000:00:14.4: >  bridge window [io  0x0d00-0xffff] (subtracti
ve decode)
[    1.051459] ACPI: PCI Interrupt Routing Table [\_SB_.PCI0._PRT]
[    1.051631] ACPI: PCI Interrupt Routing Table [\_SB_.PCI0.PB4_._PRT]
[    1.051675] ACPI: PCI Interrupt Routing Table [\_SB_.PCI0.PB6_._PRT]
[    1.051755] ACPI: PCI Interrupt Routing Table [\_SB_.PCI0.P2P_._PRT]
[    1.051836] ACPI: PCI Interrupt Routing Table [\_SB_.PCI0.AGP_._PRT]
[    1.052045]  pci0000:00: >Requesting ACPI _OSC control (0x1d)
[    1.052349]  pci0000:00: >ACPI _OSC control (0x1d) granted
[    1.056331] ACPI: PCI Interrupt Link [LNKA] (IRQs 10 11) *0, disabled.
[    1.056371] ACPI: PCI Interrupt Link [LNKB] (IRQs 10 11) *0, disabled.
[    1.056408] ACPI: PCI Interrupt Link [LNKC] (IRQs 10 11) *0, disabled.
[    1.056445] ACPI: PCI Interrupt Link [LNKD] (IRQs 10 11) *0, disabled.
[    1.056481] ACPI: PCI Interrupt Link [LNKE] (IRQs 10 11) *0, disabled.
[    1.056517] ACPI: PCI Interrupt Link [LNKF] (IRQs 10 11) *0, disabled.
[    1.056553] ACPI: PCI Interrupt Link [LNKG] (IRQs 10 11) *0, disabled.
[    1.056589] ACPI: PCI Interrupt Link [LNKH] (IRQs 10 11) *0, disabled.
[    1.056718] vgaarb: device added: PCI:0000:01:05.0,decodes=io+mem,owns=io+mem
,locks=none
[    1.056721] vgaarb: loaded
[    1.056722] vgaarb: bridge control possible 0000:01:05.0
[    1.056911] SCSI subsystem initialized
[    1.056968] libata version 3.00 loaded.
[    1.056996] ACPI: bus type usb registered
[    1.057018] usbcore: registered new interface driver usbfs
[    1.057027] usbcore: registered new interface driver hub
[    1.057050] usbcore: registered new device driver usb
[    1.057133] PCI: Using ACPI for IRQ routing
[    1.057387] PCI: pci_cache_line_size set to 64 bytes
[    1.057469] e820: reserve RAM buffer [mem 0x0009dc00-0x0009ffff]
[    1.057471] e820: reserve RAM buffer [mem 0xb7ec0000-0xb7ffffff]
[    1.057573] NetLabel: Initializing
[    1.057575] NetLabel:  domain hash size = 128
[    1.057576] NetLabel:  protocols = UNLABELED CIPSOv4
[    1.057589] NetLabel:  unlabeled traffic allowed by default
[    1.057706] hpet0: at MMIO 0xfed00000, IRQs 2, 8, 0, 0
[    1.057711] hpet0: 4 comparators, 32-bit 14.318180 MHz counter
[    1.059764] Switching to clocksource hpet
[    1.067490] AppArmor: AppArmor Filesystem Enabled
[    1.067527] pnp: PnP ACPI init
[    1.067548] ACPI: bus type pnp registered
[    1.068331] pnp 00:00: >[bus 00-ff]
[    1.068335] pnp 00:00: >[mem 0x000a0000-0x000bffff window]
[    1.068338] pnp 00:00: >[mem 0x000c0000-0x000c1fff window]
[    1.068340] pnp 00:00: >[mem 0x000c2000-0x000c3fff window]
[    1.068342] pnp 00:00: >[mem 0x000c4000-0x000c5fff window]
[    1.068344] pnp 00:00: >[mem 0x000c6000-0x000c7fff window]
[    1.068346] pnp 00:00: >[mem 0x000c8000-0x000c9fff window]
[    1.068348] pnp 00:00: >[mem 0x000ca000-0x000cbfff window]
[    1.068350] pnp 00:00: >[mem 0x000cc000-0x000cdfff window]
[    1.068352] pnp 00:00: >[mem 0x000ce000-0x000cffff window]
[    1.068354] pnp 00:00: >[mem 0x000d0000-0x000d1fff window]
[    1.068356] pnp 00:00: >[mem 0x000d2000-0x000d3fff window]
[    1.068358] pnp 00:00: >[mem 0x000d4000-0x000d5fff window]
[    1.068360] pnp 00:00: >[mem 0x000d6000-0x000d7fff window]
[    1.068365] pnp 00:00: >[mem 0x000d8000-0x000d9fff window]
[    1.068367] pnp 00:00: >[mem 0x000da000-0x000dbfff window]
[    1.068369] pnp 00:00: >[mem 0x000dc000-0x000ddfff window]
[    1.068371] pnp 00:00: >[mem 0x000de000-0x000dffff window]
[    1.068373] pnp 00:00: >[mem 0x000e0000-0x000e1fff window]
[    1.068375] pnp 00:00: >[mem 0x000e2000-0x000e3fff window]
[    1.068377] pnp 00:00: >[mem 0x000e4000-0x000e5fff window]
[    1.068379] pnp 00:00: >[mem 0x000e6000-0x000e7fff window]
[    1.068381] pnp 00:00: >[mem 0x000e8000-0x000e9fff window]
[    1.068383] pnp 00:00: >[mem 0x000ea000-0x000ebfff window]
[    1.068385] pnp 00:00: >[mem 0x000ec000-0x000edfff window]
[    1.068387] pnp 00:00: >[mem 0x000ee000-0x000effff window]
[    1.068389] pnp 00:00: >[mem 0xc8000000-0xdfffffff window]
[    1.068391] pnp 00:00: >[mem 0xf0000000-0xffffffff window]
[    1.068393] pnp 00:00: >[io  0x0cf8-0x0cff]
[    1.068396] pnp 00:00: >[io  0x0000-0x0cf7 window]
[    1.068398] pnp 00:00: >[io  0x0d00-0xffff window]
[    1.068479] pnp 00:00: >Plug and Play ACPI device, IDs PNP0a08 PNP0a03 (activ
e)
[    1.068523] pnp 00:01: >[io  0x0f50-0x0f51]
[    1.068525] pnp 00:01: >[mem 0xfec00000-0xfec00fff]
[    1.068527] pnp 00:01: >[mem 0xfee00000-0xfee00fff]
[    1.068580] system 00:01: >[io  0x0f50-0x0f51] has been reserved
[    1.068584] system 00:01: >[mem 0xfec00000-0xfec00fff] could not be reserved
[    1.068586] system 00:01: >[mem 0xfee00000-0xfee00fff] has been reserved
[    1.068590] system 00:01: >Plug and Play ACPI device, IDs PNP0c02 (active)
[    1.068956] pnp 00:02: >[io  0x0000-0x001f]
[    1.068958] pnp 00:02: >[io  0x0080-0x008f]
[    1.068960] pnp 00:02: >[io  0x00c0-0x00df]
[    1.068962] pnp 00:02: >[dma 4]
[    1.068996] pnp 00:02: >Plug and Play ACPI device, IDs PNP0200 (active)
[    1.069004] pnp 00:03: >[io  0x00f0-0x00fe]
[    1.069050] pnp 00:03: >[irq 13]
[    1.069085] pnp 00:03: >Plug and Play ACPI device, IDs PNP0c04 (active)
[    1.069095] pnp 00:04: >[io  0x0070-0x0071]
[    1.069134] pnp 00:04: >[irq 8]
[    1.069165] pnp 00:04: >Plug and Play ACPI device, IDs PNP0b00 (active)
[    1.069173] pnp 00:05: >[io  0x0061]
[    1.069207] pnp 00:05: >Plug and Play ACPI device, IDs PNP0800 (active)
[    1.069223] pnp 00:06: >[io  0x0022-0x0023]
[    1.069225] pnp 00:06: >[io  0x002e-0x002f]
[    1.069227] pnp 00:06: >[io  0x0072-0x0073]
[    1.069229] pnp 00:06: >[io  0x0080-0x0081]
[    1.069231] pnp 00:06: >[io  0x00b0-0x00b1]
[    1.069233] pnp 00:06: >[io  0x00b8]
[    1.069234] pnp 00:06: >[io  0x0092]
[    1.069236] pnp 00:06: >[io  0x0220-0x022f]
[    1.069238] pnp 00:06: >[io  0x040b]
[    1.069240] pnp 00:06: >[io  0x04d0-0x04d1]
[    1.069242] pnp 00:06: >[io  0x04d6]
[    1.069243] pnp 00:06: >[io  0x0530-0x0537]
[    1.069245] pnp 00:06: >[io  0x0c00-0x0c01]
[    1.069249] pnp 00:06: >[io  0x0c14]
[    1.069251] pnp 00:06: >[io  0x0c50-0x0c52]
[    1.069253] pnp 00:06: >[io  0x0c6c]
[    1.069255] pnp 00:06: >[io  0x0c6f]
[    1.069257] pnp 00:06: >[io  0x0cd0-0x0cd1]
[    1.069258] pnp 00:06: >[io  0x0cd2-0x0cd3]
[    1.069260] pnp 00:06: >[io  0x0cd4-0x0cd5]
[    1.069262] pnp 00:06: >[io  0x0cd6-0x0cd7]
[    1.069264] pnp 00:06: >[io  0x0cd8-0x0cdf]
[    1.069266] pnp 00:06: >[io  0x8000-0x805f]
[    1.069267] pnp 00:06: >[io  0x004e-0x004f]
[    1.069269] pnp 00:06: >[io  0x0068-0x006f]
[    1.069271] pnp 00:06: >[mem 0xff800000-0xffefffff]
[    1.069273] pnp 00:06: >[mem 0xfec10000-0xfec1001f]
[    1.069275] pnp 00:06: >[io  0x00b8]
[    1.069277] pnp 00:06: >[io  0x8100-0x81ff window]
[    1.069279] pnp 00:06: >[io  0x8200-0x82ff window]
[    1.069281] pnp 00:06: >[io  0x0f40-0x0f47]
[    1.069283] pnp 00:06: >[io  0x087f]
[    1.069346] system 00:06: >[io  0x0220-0x022f] has been reserved
[    1.069349] system 00:06: >[io  0x040b] has been reserved
[    1.069351] system 00:06: >[io  0x04d0-0x04d1] has been reserved
[    1.069354] system 00:06: >[io  0x04d6] has been reserved
[    1.069356] system 00:06: >[io  0x0530-0x0537] has been reserved
[    1.069358] system 00:06: >[io  0x0c00-0x0c01] has been reserved
[    1.069361] system 00:06: >[io  0x0c14] has been reserved
[    1.069363] system 00:06: >[io  0x0c50-0x0c52] has been reserved
[    1.069366] system 00:06: >[io  0x0c6c] has been reserved
[    1.069368] system 00:06: >[io  0x0c6f] has been reserved
[    1.069370] system 00:06: >[io  0x0cd0-0x0cd1] has been reserved
[    1.069373] system 00:06: >[io  0x0cd2-0x0cd3] has been reserved
[    1.069375] system 00:06: >[io  0x0cd4-0x0cd5] has been reserved
[    1.069378] system 00:06: >[io  0x0cd6-0x0cd7] has been reserved
[    1.069380] system 00:06: >[io  0x0cd8-0x0cdf] has been reserved
[    1.069383] system 00:06: >[io  0x8000-0x805f] has been reserved
[    1.069386] system 00:06: >[io  0x8100-0x81ff window] has been reserved
[    1.069388] system 00:06: >[io  0x8200-0x82ff window] has been reserved
[    1.069391] system 00:06: >[io  0x0f40-0x0f47] has been reserved
[    1.069393] system 00:06: >[io  0x087f] has been reserved
[    1.069397] system 00:06: >[mem 0xff800000-0xffefffff] has been reserved
[    1.069399] system 00:06: >[mem 0xfec10000-0xfec1001f] has been reserved
[    1.069402] system 00:06: >Plug and Play ACPI device, IDs PNP0c02 (active)
[    1.069499] pnp 00:07: >[mem 0x000e0000-0x000fffff]
[    1.069501] pnp 00:07: >[mem 0xffe00000-0xffffffff]
[    1.069504] pnp 00:07: >[mem 0x00000000-0xffffffffffffffff disabled]
[    1.069506] pnp 00:07: >[mem 0xfec10000-0xfec1001f]
[    1.069523] pnp 00:07: >[Firmware Bug]: [mem 0x00000000-0xffffffffffffffff di
sabled] covers only part of AMD MMCONFIG area [mem 0xe0000000-0xefffffff]; addin
g more reservations
[    1.069561] system 00:07: >[mem 0x000e0000-0x000fffff] could not be reserved
[    1.069564] system 00:07: >[mem 0xffe00000-0xffffffff] could not be reserved
[    1.069567] system 00:07: >[mem 0xfec10000-0xfec1001f] has been reserved
[    1.069570] system 00:07: >Plug and Play ACPI device, IDs PNP0c01 (active)
[    1.071702] pnp 00:08: >[io  0x0060]
[    1.071709] pnp 00:08: >[io  0x0064]
[    1.071754] pnp 00:08: >[irq 1]
[    1.071843] pnp 00:08: >Plug and Play ACPI device, IDs PNP0303 (active)
[    1.071889] pnp 00:09: >[irq 12]
[    1.071956] pnp 00:09: >Plug and Play ACPI device, IDs SYN1b22 SYN1b00 SYN000
2 PNP0f13 (active)
[    1.072038] pnp 00:0a: >[mem 0xfed00000-0xfed003ff]
[    1.072073] pnp 00:0a: >Plug and Play ACPI device, IDs PNP0103 (active)
[    1.072099] pnp: PnP ACPI: found 11 devices
[    1.072100] ACPI: ACPI bus type pnp unregistered
[    1.078445] pci 0000:00:01.0: >PCI bridge to [bus 01-01]
[    1.078451] pci 0000:00:01.0: >  bridge window [io  0x9000-0x9fff]
[    1.078455] pci 0000:00:01.0: >  bridge window [mem 0xcfd00000-0xcfefffff]
[    1.078459] pci 0000:00:01.0: >  bridge window [mem 0xd0000000-0xdfffffff 64b
it pref]
[    1.078463] pci 0000:00:04.0: >PCI bridge to [bus 03-03]
[    1.078467] pci 0000:00:04.0: >  bridge window [mem 0xf0300000-0xf03fffff]
[    1.078472] pci 0000:00:06.0: >PCI bridge to [bus 09-09]
[    1.078475] pci 0000:00:06.0: >  bridge window [mem 0xf0400000-0xf04fffff]
[    1.078480] pci 0000:00:14.4: >PCI bridge to [bus 0a-0a]
[    1.078562] pci_bus 0000:00: >resource 4 [mem 0x000a0000-0x000bffff]
[    1.078565] pci_bus 0000:00: >resource 5 [mem 0x000d4000-0x000d5fff]
[    1.078567] pci_bus 0000:00: >resource 6 [mem 0x000d6000-0x000d7fff]
[    1.078569] pci_bus 0000:00: >resource 7 [mem 0x000d8000-0x000d9fff]
[    1.078571] pci_bus 0000:00: >resource 8 [mem 0x000da000-0x000dbfff]
[    1.078573] pci_bus 0000:00: >resource 9 [mem 0x000dc000-0x000ddfff]
[    1.078575] pci_bus 0000:00: >resource 10 [mem 0x000de000-0x000dffff]
[    1.078577] pci_bus 0000:00: >resource 11 [mem 0xc8000000-0xdfffffff]
[    1.078580] pci_bus 0000:00: >resource 12 [mem 0xf0000000-0xffffffff]
[    1.078582] pci_bus 0000:00: >resource 13 [io  0x0000-0x0cf7]
[    1.078584] pci_bus 0000:00: >resource 14 [io  0x0d00-0xffff]
[    1.078586] pci_bus 0000:01: >resource 0 [io  0x9000-0x9fff]
[    1.078589] pci_bus 0000:01: >resource 1 [mem 0xcfd00000-0xcfefffff]
[    1.078591] pci_bus 0000:01: >resource 2 [mem 0xd0000000-0xdfffffff 64bit pre
f]
[    1.078593] pci_bus 0000:03: >resource 1 [mem 0xf0300000-0xf03fffff]
[    1.078596] pci_bus 0000:09: >resource 1 [mem 0xf0400000-0xf04fffff]
[    1.078599] pci_bus 0000:0a: >resource 4 [mem 0x000a0000-0x000bffff]
[    1.078601] pci_bus 0000:0a: >resource 5 [mem 0x000d4000-0x000d5fff]
[    1.078603] pci_bus 0000:0a: >resource 6 [mem 0x000d6000-0x000d7fff]
[    1.078605] pci_bus 0000:0a: >resource 7 [mem 0x000d8000-0x000d9fff]
[    1.078607] pci_bus 0000:0a: >resource 8 [mem 0x000da000-0x000dbfff]
[    1.078609] pci_bus 0000:0a: >resource 9 [mem 0x000dc000-0x000ddfff]
[    1.078612] pci_bus 0000:0a: >resource 10 [mem 0x000de000-0x000dffff]
[    1.078614] pci_bus 0000:0a: >resource 11 [mem 0xc8000000-0xdfffffff]
[    1.078616] pci_bus 0000:0a: >resource 12 [mem 0xf0000000-0xffffffff]
[    1.078618] pci_bus 0000:0a: >resource 13 [io  0x0000-0x0cf7]
[    1.078620] pci_bus 0000:0a: >resource 14 [io  0x0d00-0xffff]
[    1.078666] NET: Registered protocol family 2
[    1.078800] IP route cache hash table entries: 131072 (order: 8, 1048576 byte
s)
[    1.079824] TCP established hash table entries: 524288 (order: 11, 8388608 by
tes)
[    1.083129] TCP bind hash table entries: 65536 (order: 8, 1048576 bytes)
[    1.083532] TCP: Hash tables configured (established 524288 bind 65536)
[    1.083534] TCP: reno registered
[    1.083548] UDP hash table entries: 2048 (order: 4, 65536 bytes)
[    1.083588] UDP-Lite hash table entries: 2048 (order: 4, 65536 bytes)
[    1.083694] NET: Registered protocol family 1
[    1.083709] pci 0000:00:01.0: >MSI quirk detected; subordinate MSI disabled
[    1.083969] pci 0000:01:05.0: >Boot video device
[    1.083979] PCI: CLS 32 bytes, default 64
[    1.084375] PCI-DMA: Disabling AGP.
[    1.084455] PCI-DMA: aperture base @ ac000000 size 65536 KB
[    1.084456] PCI-DMA: using GART IOMMU.
[    1.084459] PCI-DMA: Reserving 64MB of IOMMU area in the AGP aperture
[    1.088128] LVT offset 1 assigned for vector 0x400
[    1.088142] IBS: LVT offset 1 assigned
[    1.088176] perf: AMD IBS detected (0x0000001f)
[    1.088355] audit: initializing netlink socket (disabled)
[    1.088372] type=2000 audit(1358127161.176:1): initialized
[    1.113927] HugeTLB registered 2 MB page size, pre-allocated 0 pages
[    1.116067] VFS: Disk quotas dquot_6.5.2
[    1.116118] Dquot-cache hash table entries: 512 (order 0, 4096 bytes)
[    1.116687] fuse init (API version 7.19)
[    1.116764] msgmni has been set to 7367
[    1.117102] Block layer SCSI generic (bsg) driver version 0.4 loaded (major 2
52)
[    1.117162] io scheduler noop registered
[    1.117164] io scheduler deadline registered (default)
[    1.117193] io scheduler cfq registered
[    1.117357] pcieport 0000:00:04.0: >irq 40 for MSI/MSI-X
[    1.117444] pcieport 0000:00:06.0: >irq 41 for MSI/MSI-X
[    1.117525] pcieport 0000:00:04.0: >Signaling PME through PCIe PME interrupt
[    1.117527] pci 0000:03:00.0: >Signaling PME through PCIe PME interrupt
[    1.117530] pcie_pme 0000:00:04.0:pcie01: >service driver pcie_pme loaded
[    1.117544] pcieport 0000:00:06.0: >Signaling PME through PCIe PME interrupt
[    1.117546] pci 0000:09:00.0: >Signaling PME through PCIe PME interrupt
[    1.117549] pcie_pme 0000:00:06.0:pcie01: >service driver pcie_pme loaded
[    1.117563] pci_hotplug: PCI Hot Plug PCI Core version: 0.5
[    1.117583] pciehp: PCI Express Hot Plug Controller Driver version: 0.4
[    1.119959] ACPI: AC Adapter [ADP1] (on-line)
[    1.120045] input: Power Button as /devices/LNXSYSTM:00/device:00/PNP0C0C:00/
input/input0
[    1.120050] ACPI: Power Button [PWRB]
[    1.120092] input: Lid Switch as /devices/LNXSYSTM:00/device:00/PNP0C0D:00/in
put/input1
[    1.121114] ACPI: Lid Switch [LID0]
[    1.121199] input: Sleep Button as /devices/LNXSYSTM:00/device:00/PNP0C0E:00/
input/input2
[    1.121204] ACPI: Sleep Button [SLPB]
[    1.121263] input: Power Button as /devices/LNXSYSTM:00/LNXPWRBN:00/input/inp
ut3
[    1.121266] ACPI: Power Button [PWRF]
[    1.121354] ACPI: processor limited to max C-state 1
[    1.141208] thermal LNXTHERM:00: >registered as thermal_zone0
[    1.141214] ACPI: Thermal Zone [TZS0] (31 C)
[    1.146580] thermal LNXTHERM:01: >registered as thermal_zone1
[    1.146586] ACPI: Thermal Zone [TZS1] (26 C)
[    1.146645] ACPI: Battery Slot [BAT0] (battery present)
[    1.146668] GHES: HEST is not enabled!
[    1.146773] Serial: 8250/16550 driver, 32 ports, IRQ sharing enabled
[    1.148030] Linux agpgart interface v0.103
[    1.149353] brd: module loaded
[    1.150049] loop: module loaded
[    1.150178] ahci 0000:00:11.0: >version 3.0
[    1.150343] ahci 0000:00:11.0: >AHCI 0001.0100 32 slots 2 ports 3 Gbps 0x3 im
pl SATA mode
[    1.150347] ahci 0000:00:11.0: >flags: 64bit ncq sntf ilck pm led clo pmp pio
 slum part ccc 
[    1.150790] scsi0 : ahci
[    1.150903] scsi1 : ahci
[    1.151031] ata1: SATA max UDMA/133 abar m1024@0xf0208000 port 0xf0208100 irq
 22
[    1.151034] ata2: SATA max UDMA/133 abar m1024@0xf0208000 port 0xf0208180 irq
 22
[    1.151356] Fixed MDIO Bus: probed
[    1.151397] tun: Universal TUN/TAP device driver, 1.6
[    1.151399] tun: (C) 1999-2004 Max Krasnyansky <maxk@qualcomm.com>
[    1.151466] PPP generic driver version 2.4.2
[    1.151509] ehci_hcd: USB 2.0 'Enhanced' Host Controller (EHCI) Driver
[    1.151594] ehci_hcd 0000:00:12.2: >EHCI Host Controller
[    1.151604] ehci_hcd 0000:00:12.2: >new USB bus registered, assigned bus numb
er 1

DarkNeo
Linux 4.x
Linux 4.x
Messaggi: 1071
Iscritto il: sab 4 giu 2005, 0:00
Nome Cognome: Stefano Arbitrio
Slackware: Slackware64 14
Kernel: 3.2.X
Desktop: KDE
Località: Matrix

Re: Ubuntu 12.10 Vs Ati catalist 12.6

Messaggio da DarkNeo »

Codice: Seleziona tutto

[    1.151613] ehci_hcd 0000:00:12.2: >applying AMD SB700/SB800/Hudson-2/3 EHCI 
dummy qh workaround
[    1.151644] ehci_hcd 0000:00:12.2: >debug port 1
[    1.151668] ehci_hcd 0000:00:12.2: >irq 17, io mem 0xf0208400
[    1.159996] ehci_hcd 0000:00:12.2: >USB 2.0 started, EHCI 1.00
[    1.160035] usb usb1: >New USB device found, idVendor=1d6b, idProduct=0002
[    1.160038] usb usb1: >New USB device strings: Mfr=3, Product=2, SerialNumber=1
[    1.160041] usb usb1: >Product: EHCI Host Controller
[    1.160043] usb usb1: >Manufacturer: Linux 3.5.0-17-generic ehci_hcd
[    1.160045] usb usb1: >SerialNumber: 0000:00:12.2
[    1.160196] hub 1-0:1.0: >USB hub found
[    1.160201] hub 1-0:1.0: >6 ports detected
[    1.160354] ehci_hcd 0000:00:13.2: >EHCI Host Controller
[    1.160361] ehci_hcd 0000:00:13.2: >new USB bus registered, assigned bus number 2
[    1.160370] ehci_hcd 0000:00:13.2: >applying AMD SB700/SB800/Hudson-2/3 EHCI dummy qh workaround
[    1.160394] ehci_hcd 0000:00:13.2: >debug port 1
[    1.160422] ehci_hcd 0000:00:13.2: >irq 19, io mem 0xf0208800
[    1.171936] ehci_hcd 0000:00:13.2: >USB 2.0 started, EHCI 1.00
[    1.171970] usb usb2: >New USB device found, idVendor=1d6b, idProduct=0002
[    1.171972] usb usb2: >New USB device strings: Mfr=3, Product=2, SerialNumber=1
[    1.171975] usb usb2: >Product: EHCI Host Controller
[    1.171977] usb usb2: >Manufacturer: Linux 3.5.0-17-generic ehci_hcd
[    1.171979] usb usb2: >SerialNumber: 0000:00:13.2
[    1.172134] hub 2-0:1.0: >USB hub found
[    1.172139] hub 2-0:1.0: >6 ports detected
[    1.172270] ohci_hcd: USB 1.1 'Open' Host Controller (OHCI) Driver
[    1.172357] ohci_hcd 0000:00:12.0: >OHCI Host Controller
[    1.172364] ohci_hcd 0000:00:12.0: >new USB bus registered, assigned bus number 3
[    1.172396] ohci_hcd 0000:00:12.0: >irq 16, io mem 0xf0004000
[    1.231949] usb usb3: >New USB device found, idVendor=1d6b, idProduct=0001
[    1.231954] usb usb3: >New USB device strings: Mfr=3, Product=2, SerialNumber=1
[    1.231957] usb usb3: >Product: OHCI Host Controller
[    1.231959] usb usb3: >Manufacturer: Linux 3.5.0-17-generic ohci_hcd
[    1.231961] usb usb3: >SerialNumber: 0000:00:12.0
[    1.232156] hub 3-0:1.0: >USB hub found
[    1.232196] hub 3-0:1.0: >3 ports detected
[    1.232300] ohci_hcd 0000:00:12.1: >OHCI Host Controller
[    1.232307] ohci_hcd 0000:00:12.1: >new USB bus registered, assigned bus number 4
[    1.232330] ohci_hcd 0000:00:12.1: >irq 16, io mem 0xf0005000
[    1.261583] ACPI: Battery Slot [BAT0] (battery present)
[    1.291914] usb usb4: >New USB device found, idVendor=1d6b, idProduct=0001
[    1.291919] usb usb4: >New USB device strings: Mfr=3, Product=2, SerialNumber=1
[    1.291922] usb usb4: >Product: OHCI Host Controller
[    1.291924] usb usb4: >Manufacturer: Linux 3.5.0-17-generic ohci_hcd
[    1.291926] usb usb4: >SerialNumber: 0000:00:12.1
[    1.292082] hub 4-0:1.0: >USB hub found
[    1.292125] hub 4-0:1.0: >3 ports detected
[    1.292228] ohci_hcd 0000:00:13.0: >OHCI Host Controller
[    1.292234] ohci_hcd 0000:00:13.0: >new USB bus registered, assigned bus number 5
[    1.292267] ohci_hcd 0000:00:13.0: >irq 18, io mem 0xf0006000
[    1.313264] Freeing initrd memory: 14868k freed
[    1.351889] usb usb5: >New USB device found, idVendor=1d6b, idProduct=0001
[    1.351893] usb usb5: >New USB device strings: Mfr=3, Product=2, SerialNumber=1
[    1.351896] usb usb5: >Product: OHCI Host Controller
[    1.351898] usb usb5: >Manufacturer: Linux 3.5.0-17-generic ohci_hcd
[    1.351900] usb usb5: >SerialNumber: 0000:00:13.0
[    1.352137] hub 5-0:1.0: >USB hub found
[    1.352179] hub 5-0:1.0: >3 ports detected
[    1.352331] ohci_hcd 0000:00:13.1: >OHCI Host Controller
[    1.352338] ohci_hcd 0000:00:13.1: >new USB bus registered, assigned bus number 6
[    1.352362] ohci_hcd 0000:00:13.1: >irq 18, io mem 0xf0007000
[    1.411876] usb usb6: >New USB device found, idVendor=1d6b, idProduct=0001
[    1.411880] usb usb6: >New USB device strings: Mfr=3, Product=2, SerialNumber=1
[    1.411883] usb usb6: >Product: OHCI Host Controller
[    1.411885] usb usb6: >Manufacturer: Linux 3.5.0-17-generic ohci_hcd
[    1.411887] usb usb6: >SerialNumber: 0000:00:13.1
[    1.412170] hub 6-0:1.0: >USB hub found
[    1.412212] hub 6-0:1.0: >3 ports detected
[    1.412326] uhci_hcd: USB Universal Host Controller Interface driver
[    1.412421] usbcore: registered new interface driver libusual
[    1.412460] i8042: PNP: PS/2 Controller [PNP0303:KBD0,PNP0f13:PS2M] at 0x60,0x64 irq 1,12
[    1.422563] i8042: Detected active multiplexing controller, rev 1.1
[    1.429133] serio: i8042 KBD port at 0x60,0x64 irq 1
[    1.429140] serio: i8042 AUX0 port at 0x60,0x64 irq 12
[    1.429167] serio: i8042 AUX1 port at 0x60,0x64 irq 12
[    1.429188] serio: i8042 AUX2 port at 0x60,0x64 irq 12
[    1.429206] serio: i8042 AUX3 port at 0x60,0x64 irq 12
[    1.429369] mousedev: PS/2 mouse device common for all mice
[    1.429697] rtc_cmos 00:04: >RTC can wake from S4
[    1.429861] rtc_cmos 00:04: >rtc core: registered rtc_cmos as rtc0
[    1.429887] rtc0: alarms up to one month, 114 bytes nvram, hpet irqs
[    1.429977] device-mapper: uevent: version 1.0.3
[    1.430181] device-mapper: ioctl: 4.22.0-ioctl (2011-10-19) initialised: dm-devel@redhat.com
[    1.430191] cpuidle: using governor ladder
[    1.430192] cpuidle: using governor menu
[    1.430194] EFI Variables Facility v0.08 2004-May-17
[    1.430642] ashmem: initialized
[    1.430781] TCP: cubic registered
[    1.430890] NET: Registered protocol family 10
[    1.431094] NET: Registered protocol family 17
[    1.431104] Key type dns_resolver registered
[    1.431335] PM: Hibernation image not present or could not be loaded.
[    1.431347] registered taskstats version 1
[    1.434328] Key type trusted registered
[    1.436854] Key type encrypted registered
[    1.439780]   Magic number: 13:599:508
[    1.439840] pci 0000:00:01.0: >hash matches
[    1.439910] rtc_cmos 00:04: >setting system clock to 2013-01-14 01:32:43 UTC (1358127163)
[    1.439985] powernow-k8: Found 1 AMD Athlon(tm) II Dual-Core M300 (2 cpu cores) (version 2.20.00)
[    1.440030] powernow-k8:    0 : pstate 0 (2000 MHz)
[    1.440032] powernow-k8:    1 : pstate 1 (1400 MHz)
[    1.440033] powernow-k8:    2 : pstate 2 (800 MHz)
[    1.440201] BIOS EDD facility v0.16 2004-Jun-25, 0 devices found
[    1.440202] EDD information not available.
[    1.451073] input: AT Translated Set 2 keyboard as /devices/platform/i8042/serio0/input/input4
[    1.471835] usb 1-1: >new high-speed USB device number 2 using ehci_hcd
[    1.608160] usb 1-1: >New USB device found, idVendor=05ac, idProduct=1293
[    1.608165] usb 1-1: >New USB device strings: Mfr=1, Product=2, SerialNumber=3
[    1.608168] usb 1-1: >Product: iPod
[    1.608170] usb 1-1: >Manufacturer: Apple Inc.
[    1.608172] usb 1-1: >SerialNumber: 01265dfec3e5d4a986a889c34bb2400cc8d28eeb
[    1.639707] ata1: SATA link up 3.0 Gbps (SStatus 123 SControl 300)
[    1.640814] ata1.00: ATA-8: Hitachi HTS545050B9A300, PB4OC60F, max UDMA/133
[    1.640819] ata1.00: 976773168 sectors, multi 16: LBA48 NCQ (depth 31/32), AA
[    1.642149] ata1.00: configured for UDMA/133
[    1.642426] scsi 0:0:0:0: >Direct-Access     ATA      Hitachi HTS54505 PB4O PQ: 0 ANSI: 5
[    1.642611] sd 0:0:0:0: >Attached scsi generic sg0 type 0
[    1.642621] sd 0:0:0:0: >[sda] 976773168 512-byte logical blocks: (500 GB/465 GiB)
[    1.642665] sd 0:0:0:0: >[sda] Write Protect is off
[    1.642668] sd 0:0:0:0: >[sda] Mode Sense: 00 3a 00 00
[    1.642733] sd 0:0:0:0: >[sda] Write cache: enabled, read cache: enabled, doesn't support DPO or FUA
[    1.644550] ata2: SATA link up 1.5 Gbps (SStatus 113 SControl 300)
[    1.658512] ata2.00: ATAPI: Optiarc DVD RW AD-7580S, FX20, max UDMA/100
[    1.668293]  sda: sda1 sda2 sda3
[    1.668590] sd 0:0:0:0: >[sda] Attached SCSI disk
[    1.672275] ata2.00: configured for UDMA/100
[    1.675513] scsi 1:0:0:0: >CD-ROM            Optiarc  DVD RW AD-7580S  FX20 PQ: 0 ANSI: 5
[    1.681722] sr0: scsi3-mmc drive: 24x/24x writer dvd-ram cd/rw xa/form2 cdda tray
[    1.681728] cdrom: Uniform CD-ROM driver Revision: 3.20
[    1.681854] sr 1:0:0:0: >Attached scsi CD-ROM sr0
[    1.681932] sr 1:0:0:0: >Attached scsi generic sg1 type 5
[    1.708146] Freeing unused kernel memory: 928k freed
[    1.708439] Write protecting the kernel read-only data: 12288k
[    1.713918] Freeing unused kernel memory: 1472k freed
[    1.718405] Freeing unused kernel memory: 1124k freed
[    1.746460] udevd[93]: starting version 175
[    1.759637] usb 2-5: >new high-speed USB device number 2 using ehci_hcd
[    1.857165] tg3.c:v3.123 (March 21, 2012)
[    1.907915] tg3 0000:03:00.0: >eth0: Tigon3 [partno(BCM95784M) rev 5784100] (PCI Express) MAC address 00:26:2d:71:2c:d1
[    1.907922] tg3 0000:03:00.0: >eth0: attached PHY is 5784 (10/100/1000Base-T Ethernet) (WireSpeed[1], EEE[0])
[    1.907925] tg3 0000:03:00.0: >eth0: RXcsums[1] LinkChgREG[0] MIirq[0] ASF[0] TSOcap[1]
[    1.907928] tg3 0000:03:00.0: >eth0: dma_rwctrl[76180000] dma_mask[64-bit]
[    1.929937] usb 2-5: >New USB device found, idVendor=04f2, idProduct=b044
[    1.929942] usb 2-5: >New USB device strings: Mfr=2, Product=1, SerialNumber=3
[    1.929945] usb 2-5: >Product: CNF7017
[    1.929948] usb 2-5: >Manufacturer: Chicony Electronics Co., Ltd.
[    1.929950] usb 2-5: >SerialNumber: SN0001
[    2.043457] usb 2-6: >new high-speed USB device number 3 using ehci_hcd
[    2.083478] Refined TSC clocksource calibration: 1994.999 MHz.
[    2.083486] Switching to clocksource tsc
[    2.186874] usb 2-6: >New USB device found, idVendor=0bda, idProduct=0159
[    2.186879] usb 2-6: >New USB device strings: Mfr=1, Product=2, SerialNumber=3
[    2.186883] usb 2-6: >Product: USB2.0-CRW
[    2.186885] usb 2-6: >Manufacturer: Generic
[    2.186887] usb 2-6: >SerialNumber: 20071114173400000
[    2.193585] Initializing USB Mass Storage driver...
[    2.193668] usbcore: registered new interface driver usb-storage
[    2.193670] USB Mass Storage support registered.
[    2.194071] usbcore: registered new interface driver uas
[    2.205565] scsi2 : usb-storage 2-6:1.0
[    2.205678] usbcore: registered new interface driver ums-realtek
[    2.221171] EXT4-fs (sda3): INFO: recovery required on readonly filesystem
[    2.221177] EXT4-fs (sda3): write access will be enabled during recovery
[    3.110926] EXT4-fs (sda3): recovery complete
[    3.118700] EXT4-fs (sda3): mounted filesystem with ordered data mode. Opts: (null)
[    3.205438] scsi 2:0:0:0: >Direct-Access     Generic- Multi-Card       1.00 PQ: 0 ANSI: 0 CCS
[    3.206353] sd 2:0:0:0: >Attached scsi generic sg2 type 0
[    3.863327] sd 2:0:0:0: >[sdb] 3805184 512-byte logical blocks: (1.94 GB/1.81 GiB)
[    3.864196] sd 2:0:0:0: >[sdb] Write Protect is off
[    3.864203] sd 2:0:0:0: >[sdb] Mode Sense: 03 00 00 00
[    3.865069] sd 2:0:0:0: >[sdb] No Caching mode page present
[    3.865075] sd 2:0:0:0: >[sdb] Assuming drive cache: write through
[    3.868805] sd 2:0:0:0: >[sdb] No Caching mode page present
[    3.868811] sd 2:0:0:0: >[sdb] Assuming drive cache: write through
[    3.872812]  sdb: unknown partition table
[    3.875563] sd 2:0:0:0: >[sdb] No Caching mode page present
[    3.875569] sd 2:0:0:0: >[sdb] Assuming drive cache: write through
[    3.875572] sd 2:0:0:0: >[sdb] Attached SCSI removable disk
[    6.137060] IPv6: ADDRCONF(NETDEV_UP): eth0: link is not ready
[    6.470907] udevd[345]: starting version 175
[    7.260822] EXT4-fs (sda3): re-mounted. Opts: errors=remount-ro
[    7.826710] lp: driver loaded but no devices found
[    9.702037] wmi: Mapper loaded
[    9.779068] acpi device:38: >registered as cooling_device2
[    9.779204] ACPI: Video Device [VGA1] (multi-head: yes  rom: no  post: no)
[    9.779392] input: Video Bus as /devices/LNXSYSTM:00/device:00/PNP0A08:00/device:35/LNXVIDEO:01/input/input5
[    9.986450] shpchp: Standard Hot Plug PCI Controller Driver version: 0.4
[   10.155899] MCE: In-kernel MCE decoding enabled.
[   10.188833] EDAC MC: Ver: 2.1.0
[   10.212892] piix4_smbus 0000:00:14.0: >SMBus Host Controller at 0x8040, revision 0
[   10.217177] microcode: CPU0: patch_level=0x01000098
[   10.222773] AMD64 EDAC driver v3.4.0
[   10.222832] EDAC amd64: DRAM ECC disabled.
[   10.222845] EDAC amd64: ECC disabled in the BIOS or no ECC capability, module will not load.
[   10.222845]  Either enable ECC checking or force module loading by setting 'ecc_enable_override'.
[   10.222845]  (Note that use of the override may cause unknown side effects.)
[   10.282760] sp5100_tco: SP5100 TCO WatchDog Timer Driver v0.01
[   10.282900] sp5100_tco: mmio address 0xfec000f0 already in use
[   10.291878] acer_wmi: Acer Laptop ACPI-WMI Extras
[   10.295235] acer_wmi: Function bitmap for Communication Device: 0x3
[   10.295760] acer_wmi: Brightness must be controlled by acpi video driver
[   10.295985] input: Acer BMA150 accelerometer as /devices/virtual/input/input6
[   10.324028] microcode: failed to load file amd-ucode/microcode_amd.bin
[   10.324043] microcode: CPU1: patch_level=0x01000098
[   10.326341] microcode: failed to load file amd-ucode/microcode_amd.bin
[   10.326736] microcode: Microcode Update Driver: v2.00 <tigran@aivazian.fsnet.co.uk>, Peter Oruba
[   10.345343] cfg80211: Calling CRDA to update world regulatory domain
[   10.810832] kvm: Nested Virtualization enabled
[   10.810838] kvm: Nested Paging enabled
[   11.240605] psmouse serio2: >synaptics: Touchpad model: 1, fw: 7.2, id: 0x1c0b1, caps: 0xd04733/0xa44000/0xa0000
[   11.245060] Linux video capture interface: v2.00
[   11.293681] input: SynPS/2 Synaptics TouchPad as /devices/platform/i8042/serio2/input/input7
[   11.427331] uvcvideo: Found UVC 1.00 device CNF7017 (04f2:b044)
[   11.483119] input: CNF7017 as /devices/pci0000:00/0000:00:13.2/usb2/2-5/2-5:1.0/input/input8
[   11.483342] usbcore: registered new interface driver uvcvideo
[   11.483344] USB Video Class driver (1.1.1)
[   11.497593] cfg80211: World regulatory domain updated:
[   11.497599] cfg80211:   (start_freq - end_freq @ bandwidth), (max_antenna_gain, max_eirp)
[   11.497601] cfg80211:   (2402000 KHz - 2472000 KHz @ 40000 KHz), (300 mBi, 2000 mBm)
[   11.497604] cfg80211:   (2457000 KHz - 2482000 KHz @ 20000 KHz), (300 mBi, 2000 mBm)
[   11.497605] cfg80211:   (2474000 KHz - 2494000 KHz @ 20000 KHz), (300 mBi, 2000 mBm)
[   11.497607] cfg80211:   (5170000 KHz - 5250000 KHz @ 40000 KHz), (300 mBi, 2000 mBm)
[   11.497609] cfg80211:   (5735000 KHz - 5835000 KHz @ 40000 KHz), (300 mBi, 2000 mBm)
[   12.056282] ath: EEPROM regdomain: 0x65
[   12.056287] ath: EEPROM indicates we should expect a direct regpair map
[   12.056290] ath: Country alpha2 being used: 00
[   12.056291] ath: Regpair used: 0x65
[   12.056295] cfg80211: Updating information on frequency 2412 MHz for a 20 MHz width channel with regulatory rule:
[   12.056297] cfg80211: 2402000 KHz - 2472000 KHz @ 40000 KHz), (N/A mBi, 2000 mBm)
[   12.056299] cfg80211: Updating information on frequency 2417 MHz for a 20 MHz width channel with regulatory rule:
[   12.056300] cfg80211: 2402000 KHz - 2472000 KHz @ 40000 KHz), (N/A mBi, 2000 mBm)
[   12.056302] cfg80211: Updating information on frequency 2422 MHz for a 20 MHz width channel with regulatory rule:
[   12.056304] cfg80211: 2402000 KHz - 2472000 KHz @ 40000 KHz), (N/A mBi, 2000 mBm)
[   12.056305] cfg80211: Updating information on frequency 2427 MHz for a 20 MHz width channel with regulatory rule:
[   12.056307] cfg80211: 2402000 KHz - 2472000 KHz @ 40000 KHz), (N/A mBi, 2000 mBm)
[   12.056308] cfg80211: Updating information on frequency 2432 MHz for a 20 MHz width channel with regulatory rule:
[   12.056310] cfg80211: 2402000 KHz - 2472000 KHz @ 40000 KHz), (N/A mBi, 2000 mBm)
[   12.056311] cfg80211: Updating information on frequency 2437 MHz for a 20 MHz width channel with regulatory rule:
[   12.056313] cfg80211: 2402000 KHz - 2472000 KHz @ 40000 KHz), (N/A mBi, 2000 mBm)
[   12.056314] cfg80211: Updating information on frequency 2442 MHz for a 20 MHz width channel with regulatory rule:
[   12.056316] cfg80211: 2402000 KHz - 2472000 KHz @ 40000 KHz), (N/A mBi, 2000 mBm)
[   12.056317] cfg80211: Updating information on frequency 2447 MHz for a 20 MHz width channel with regulatory rule:
[   12.056319] cfg80211: 2402000 KHz - 2472000 KHz @ 40000 KHz), (N/A mBi, 2000 mBm)
[   12.056320] cfg80211: Updating information on frequency 2452 MHz for a 20 MHz width channel with regulatory rule:
[   12.056322] cfg80211: 2402000 KHz - 2472000 KHz @ 40000 KHz), (N/A mBi, 2000 mBm)
[   12.056324] cfg80211: Updating information on frequency 2457 MHz for a 20 MHz width channel with regulatory rule:
[   12.056325] cfg80211: 2402000 KHz - 2472000 KHz @ 40000 KHz), (N/A mBi, 2000 mBm)
[   12.056327] cfg80211: Updating information on frequency 2462 MHz for a 20 MHz width channel with regulatory rule:
[   12.056328] cfg80211: 2402000 KHz - 2472000 KHz @ 40000 KHz), (N/A mBi, 2000 mBm)
[   12.056330] cfg80211: Updating information on frequency 2467 MHz for a 20 MHz width channel with regulatory rule:
[   12.056332] cfg80211: 2457000 KHz - 2482000 KHz @ 40000 KHz), (N/A mBi, 2000 mBm)
[   12.056333] cfg80211: Updating information on frequency 2472 MHz for a 20 MHz width channel with regulatory rule:
[   12.056335] cfg80211: 2457000 KHz - 2482000 KHz @ 40000 KHz), (N/A mBi, 2000 mBm)
[   12.056336] cfg80211: Disabling freq 2484 MHz as custom regd has no rule that fits a 20 MHz wide channel
[   12.059540] cfg80211: Ignoring regulatory request Set by core since the driver uses its own custom regulatory domain
[   12.060132] snd_hda_intel 0000:00:14.2: >power state changed by ACPI to D0
[   12.060137] snd_hda_intel 0000:00:14.2: >power state changed by ACPI to D0
[   12.139917] input: HDA ATI SB Line as /devices/pci0000:00/0000:00:14.2/sound/card0/input9
[   12.140013] input: HDA ATI SB Mic as /devices/pci0000:00/0000:00:14.2/sound/card0/input10
[   12.140072] input: HDA ATI SB Headphone as /devices/pci0000:00/0000:00:14.2/sound/card0/input11
[   12.140526] snd_hda_intel 0000:01:05.1: >setting latency timer to 64
[   12.151277] ieee80211 phy0: >Selected rate control algorithm 'ath9k_rate_control'
[   12.151601] Registered led device: ath9k-phy0
[   12.151608] ieee80211 phy0: >Atheros AR9280 Rev:2 mem=0xffffc90001aa0000, irq=18
[   12.901680] type=1400 audit(1358123574.963:2): apparmor="STATUS" operation="profile_load" name="/sbin/dhclient" pid=641 comm="apparmor_parser"
[   12.901719] type=1400 audit(1358123574.963:3): apparmor="STATUS" operation="profile_replace" name="/sbin/dhclient" pid=640 comm="apparmor_parser"
[   12.902252] type=1400 audit(1358123574.963:4): apparmor="STATUS" operation="profile_load" name="/usr/lib/NetworkManager/nm-dhcp-client.action" pid=641 comm="apparmor_parser"
[   12.902294] type=1400 audit(1358123574.963:5): apparmor="STATUS" operation="profile_replace" name="/usr/lib/NetworkManager/nm-dhcp-client.action" pid=640 comm="apparmor_parser"
[   12.902552] type=1400 audit(1358123574.963:6): apparmor="STATUS" operation="profile_load" name="/usr/lib/connman/scripts/dhclient-script" pid=641 comm="apparmor_parser"
[   12.902600] type=1400 audit(1358123574.963:7): apparmor="STATUS" operation="profile_replace" name="/usr/lib/connman/scripts/dhclient-script" pid=640 comm="apparmor_parser"
[   12.903241] type=1400 audit(1358123574.963:8): apparmor="STATUS" operation="profile_replace" name="/sbin/dhclient" pid=681 comm="apparmor_parser"
[   12.903814] type=1400 audit(1358123574.963:9): apparmor="STATUS" operation="profile_replace" name="/usr/lib/NetworkManager/nm-dhcp-client.action" pid=681 comm="apparmor_parser"
[   12.904116] type=1400 audit(1358123574.963:10): apparmor="STATUS" operation="profile_replace" name="/usr/lib/connman/scripts/dhclient-script" pid=681 comm="apparmor_parser"
[   14.300735] init: failsafe main process (708) killed by TERM signal
[   14.817727] ppdev: user-space parallel port driver
[   15.113544] Bluetooth: Core ver 2.16
[   15.113573] NET: Registered protocol family 31
[   15.113574] Bluetooth: HCI device and connection manager initialized
[   15.113577] Bluetooth: HCI socket layer initialized
[   15.113579] Bluetooth: L2CAP socket layer initialized
[   15.113583] Bluetooth: SCO socket layer initialized
[   15.416223] Bluetooth: RFCOMM TTY layer initialized
[   15.416230] Bluetooth: RFCOMM socket layer initialized
[   15.416232] Bluetooth: RFCOMM ver 1.11
[   15.523932] Bluetooth: BNEP (Ethernet Emulation) ver 1.3
[   15.523937] Bluetooth: BNEP filters: protocol multicast
[   15.897758] type=1400 audit(1358123577.959:11): apparmor="STATUS" operation="profile_load" name="/usr/lib/cups/backend/cups-pdf" pid=784 comm="apparmor_parser"
[   18.011439] audit_printk_skb: 48 callbacks suppressed
[   18.011443] type=1400 audit(1358123580.075:28): apparmor="STATUS" operation="profile_load" name="/usr/bin/evince" pid=837 comm="apparmor_parser"
[   18.018340] type=1400 audit(1358123580.079:29): apparmor="STATUS" operation="profile_load" name="/usr/bin/evince//sanitized_helper" pid=837 comm="apparmor_parser"
[   18.019582] type=1400 audit(1358123580.083:30): apparmor="STATUS" operation="profile_load" name="/usr/bin/evince-previewer" pid=837 comm="apparmor_parser"
[   18.023514] type=1400 audit(1358123580.087:31): apparmor="STATUS" operation="profile_load" name="/usr/bin/evince-previewer//sanitized_helper" pid=837 comm="apparmor_parser"
[   18.024366] type=1400 audit(1358123580.087:32): apparmor="STATUS" operation="profile_load" name="/usr/bin/evince-thumbnailer" pid=837 comm="apparmor_parser"
[   18.027051] type=1400 audit(1358123580.091:33): apparmor="STATUS" operation="profile_load" name="/usr/bin/evince-thumbnailer//sanitized_helper" pid=837 comm="apparmor_parser"
[   21.224335] tg3 0000:03:00.0: >irq 42 for MSI/MSI-X
[   21.345401] IPv6: ADDRCONF(NETDEV_UP): eth0: link is not ready
[   21.345779] IPv6: ADDRCONF(NETDEV_UP): eth0: link is not ready
[   21.368965] IPv6: ADDRCONF(NETDEV_UP): wlan0: link is not ready
[   21.369324] IPv6: ADDRCONF(NETDEV_UP): wlan0: link is not ready
[   24.275719] wlan0: authenticate with 00:90:a2:43:e1:74
[   24.295236] wlan0: send auth to 00:90:a2:43:e1:74 (try 1/3)
[   24.297838] wlan0: authenticated
[   24.311122] wlan0: associate with 00:90:a2:43:e1:74 (try 1/3)
[   24.314686] wlan0: RX AssocResp from 00:90:a2:43:e1:74 (capab=0x431 status=0 aid=4)
[   24.314787] wlan0: associated
[   24.315369] IPv6: ADDRCONF(NETDEV_CHANGE): wlan0: link becomes ready
[   24.315455] cfg80211: Calling CRDA for country: IT
[   24.320185] cfg80211: Updating information on frequency 2412 MHz for a 20 MHz width channel with regulatory rule:
[   24.320190] cfg80211: 2402000 KHz - 2482000 KHz @ 40000 KHz), (N/A mBi, 2000 mBm)
[   24.320192] cfg80211: Updating information on frequency 2417 MHz for a 20 MHz width channel with regulatory rule:
[   24.320194] cfg80211: 2402000 KHz - 2482000 KHz @ 40000 KHz), (N/A mBi, 2000 mBm)
[   24.320196] cfg80211: Updating information on frequency 2422 MHz for a 20 MHz width channel with regulatory rule:
[   24.320198] cfg80211: 2402000 KHz - 2482000 KHz @ 40000 KHz), (N/A mBi, 2000 mBm)
[   24.320199] cfg80211: Updating information on frequency 2427 MHz for a 20 MHz width channel with regulatory rule:
[   24.320201] cfg80211: 2402000 KHz - 2482000 KHz @ 40000 KHz), (N/A mBi, 2000 mBm)
[   24.320202] cfg80211: Updating information on frequency 2432 MHz for a 20 MHz width channel with regulatory rule:
[   24.320204] cfg80211: 2402000 KHz - 2482000 KHz @ 40000 KHz), (N/A mBi, 2000 mBm)
[   24.320205] cfg80211: Updating information on frequency 2437 MHz for a 20 MHz width channel with regulatory rule:
[   24.320207] cfg80211: 2402000 KHz - 2482000 KHz @ 40000 KHz), (N/A mBi, 2000 mBm)
[   24.320208] cfg80211: Updating information on frequency 2442 MHz for a 20 MHz width channel with regulatory rule:
[   24.320210] cfg80211: 2402000 KHz - 2482000 KHz @ 40000 KHz), (N/A mBi, 2000 mBm)
[   24.320211] cfg80211: Updating information on frequency 2447 MHz for a 20 MHz width channel with regulatory rule:
[   24.320213] cfg80211: 2402000 KHz - 2482000 KHz @ 40000 KHz), (N/A mBi, 2000 mBm)
[   24.320215] cfg80211: Updating information on frequency 2452 MHz for a 20 MHz width channel with regulatory rule:
[   24.320216] cfg80211: 2402000 KHz - 2482000 KHz @ 40000 KHz), (N/A mBi, 2000 mBm)
[   24.320218] cfg80211: Updating information on frequency 2457 MHz for a 20 MHz width channel with regulatory rule:
[   24.320219] cfg80211: 2402000 KHz - 2482000 KHz @ 40000 KHz), (N/A mBi, 2000 mBm)
[   24.320221] cfg80211: Updating information on frequency 2462 MHz for a 20 MHz width channel with regulatory rule:
[   24.320222] cfg80211: 2402000 KHz - 2482000 KHz @ 40000 KHz), (N/A mBi, 2000 mBm)
[   24.320224] cfg80211: Updating information on frequency 2467 MHz for a 20 MHz width channel with regulatory rule:
[   24.320225] cfg80211: 2402000 KHz - 2482000 KHz @ 40000 KHz), (N/A mBi, 2000 mBm)
[   24.320227] cfg80211: Updating information on frequency 2472 MHz for a 20 MHz width channel with regulatory rule:
[   24.320228] cfg80211: 2402000 KHz - 2482000 KHz @ 40000 KHz), (N/A mBi, 2000 mBm)
[   24.320230] cfg80211: Disabling freq 2484 MHz
[   24.320234] ath: regdomain 0x817c updated by CountryIE
[   24.320236] ath: EEPROM regdomain: 0x817c
[   24.320237] ath: EEPROM indicates we should expect a country code
[   24.320238] ath: doing EEPROM country->regdmn map search
[   24.320239] ath: country maps to regdmn code: 0x37
[   24.320241] ath: Country alpha2 being used: IT
[   24.320242] ath: Regpair used: 0x37
[   24.320287] cfg80211: Regulatory domain changed to country: IT
[   24.320289] cfg80211:   (start_freq - end_freq @ bandwidth), (max_antenna_gain, max_eirp)
[   24.320290] cfg80211:   (2402000 KHz - 2482000 KHz @ 40000 KHz), (N/A, 2000 mBm)
[   24.320292] cfg80211:   (5170000 KHz - 5250000 KHz @ 40000 KHz), (N/A, 2000 mBm)
[   24.320293] cfg80211:   (5250000 KHz - 5330000 KHz @ 40000 KHz), (N/A, 2000 mBm)
[   24.320295] cfg80211:   (5490000 KHz - 5710000 KHz @ 40000 KHz), (N/A, 2700 mBm)
[   25.613470] vesafb: mode is 1024x768x32, linelength=4096, pages=0
[   25.613475] vesafb: scrolling: redraw
[   25.613478] vesafb: Truecolor: size=0:8:8:8, shift=0:16:8:0
[   25.613984] vesafb: framebuffer at 0xd0000000, mapped to 0xffffc90001d00000, using 3072k, total 3072k
[   25.615153] Console: switching to colour frame buffer device 128x48
[   25.615182] fb0: VESA VGA frame buffer device
[   25.683808] init: plymouth-splash main process (1178) terminated with status 1
[   37.749324] tg3 0000:03:00.0: >eth0: Link is up at 10 Mbps, full duplex
[   37.749330] tg3 0000:03:00.0: >eth0: Flow control is off for TX and off for RX
[   37.749888] IPv6: ADDRCONF(NETDEV_CHANGE): eth0: link becomes ready
[   99.049620] compiz[2047]: segfault at 30 ip 00007f874fde9576 sp 00007fff0c1b4df0 error 4 in libgrid.so[7f874fdd7000+27000]
[  109.334780] update-notifier[2391]: segfault at 87 ip 00007f4b0c3c5d21 sp 00007fff264187c0 error 4 in libgobject-2.0.so.0.3400.1[7f4b0c3b1000+4d000]
[  110.320751] tg3 0000:03:00.0: >eth0: Link is down
[  111.943228] tg3 0000:03:00.0: >eth0: Link is up at 100 Mbps, full duplex
[  111.943241] tg3 0000:03:00.0: >eth0: Flow control is off for TX and off for RX
[  124.525289] compiz[2395]: segfault at 30 ip 00007fe09097e576 sp 00007fffb2bfc880 error 4 in libgrid.so[7fe09096c000+27000]
[  130.867886] tg3 0000:03:00.0: >eth0: Link is down
[  135.094689] IPv6: ADDRCONF(NETDEV_UP): eth0: link is not ready
[  518.176532] usb 1-1: >USB disconnect, device number 2
[  524.485187] type=1400 audit(1358124086.068:34): apparmor="DENIED" operation="capable" parent=1 profile="/usr/lib/telepathy/mission-control-5" pid=3099 comm="mission-control" pid=3099 comm="mission-control" capability=23  capname="sys_nice"

Avatar utente
414N
Iper Master
Iper Master
Messaggi: 2922
Iscritto il: mer 13 feb 2008, 16:19
Slackware: 15.0
Kernel: 5.15.19
Desktop: KDE5
Località: Bulagna
Contatta:

Re: Ubuntu 12.10 Vs Ati catalist 12.6

Messaggio da 414N »

Bastava un semplice

Codice: Seleziona tutto

lsmod | grep fglrx
Comunque, il dmesg che hai riportato non contiene nessuna riga relativa ad fglrx. Controlla anche con lsmod e, se non viene riportato, hai trovato il tuo problema.

DarkNeo
Linux 4.x
Linux 4.x
Messaggi: 1071
Iscritto il: sab 4 giu 2005, 0:00
Nome Cognome: Stefano Arbitrio
Slackware: Slackware64 14
Kernel: 3.2.X
Desktop: KDE
Località: Matrix

Re: Ubuntu 12.10 Vs Ati catalist 12.6

Messaggio da DarkNeo »

Allora lsmod |grep fglrx non riporta nulla, quindi è questo il mio problema, ma quando do modprobe fglrx il risultato è questo:

Codice: Seleziona tutto

root@DarkNeo:/home/neo# modprobe fglrx
FATAL: Module fglrx not found.

DarkNeo
Linux 4.x
Linux 4.x
Messaggi: 1071
Iscritto il: sab 4 giu 2005, 0:00
Nome Cognome: Stefano Arbitrio
Slackware: Slackware64 14
Kernel: 3.2.X
Desktop: KDE
Località: Matrix

Re: Ubuntu 12.10 Vs Ati catalist 12.6

Messaggio da DarkNeo »

Ho provato a disinstallare i driver installati con apt-get dal repository consigliato e ho installato i driver scaricati dal sito ati ma da questo alla compilazione e non capisco se è un errore

Codice: Seleziona tutto

root@DarkNeo:/home/neo/Scaricati# dpkg -i *.deb
Selezionato il pacchetto fglrx non precedentemente selezionato.
(Lettura del database... 171470 file e directory attualmente installati.)
Estrazione di fglrx (da fglrx_8.970-0ubuntu1_amd64.deb)...
Selezionato il pacchetto fglrx-amdcccle non precedentemente selezionato.
Estrazione di fglrx-amdcccle (da fglrx-amdcccle_8.970-0ubuntu1_amd64.deb)...
Estrazione di fglrx-dev (da fglrx-dev_8.970-0ubuntu1_amd64.deb)...
Configurazione di fglrx (2:8.970-0ubuntu1)...
update-alternatives: viene usato /usr/lib/fglrx/ld.so.conf per fornire /etc/ld.so.conf.d/x86_64-linux-gnu_GL.conf (x86_64-linux-gnu_gl_conf) in modalità automatica
update-alternatives: attenzione: saltata la creazione di /etc/OpenCL/vendors/amdocl32.icd poiché il file /usr/lib/fglrx/etc/OpenCL/vendors/amdocl32.icd associato (del gruppo x86_64-linux-gnu_gl_conf) non esiste
update-alternatives: attenzione: viene forzata l'installazione dell'alternativa /usr/lib/fglrx/ld.so.conf poiché il gruppo x86_64-linux-gnu_gl_conf è danneggiato
update-alternatives: attenzione: saltata la creazione di /etc/OpenCL/vendors/amdocl32.icd poiché il file /usr/lib/fglrx/etc/OpenCL/vendors/amdocl32.icd associato (del gruppo x86_64-linux-gnu_gl_conf) non esiste
update-alternatives: viene usato /usr/lib/fglrx/alt_ld.so.conf per fornire /etc/ld.so.conf.d/i386-linux-gnu_GL.conf (i386-linux-gnu_gl_conf) in modalità automatica
update-initramfs: deferring update (trigger activated)
Loading new fglrx-8.970 DKMS files...
First Installation: checking all kernels...
Building only for 3.5.0-17-generic
Building for architecture x86_64
Module build for the currently running kernel was skipped since the
kernel source for this kernel does not seem to be installed.
update-initramfs: deferring update (trigger activated)
Elaborazione dei trigger per ureadahead...
Elaborazione dei trigger per bamfdaemon...
Rebuilding /usr/share/applications/bamf.index...
Configurazione di fglrx-amdcccle (2:8.970-0ubuntu1)...
Configurazione di fglrx-dev (2:8.970-0ubuntu1)...
Elaborazione dei trigger per initramfs-tools...
update-initramfs: Generating /boot/initrd.img-3.5.0-17-generic
Elaborazione dei trigger per libc-bin...
ldconfig deferred processing now taking place
root@DarkNeo:/home/neo/Scaricati# modprobe fglrx
FATAL: Module fglrx not found.

Avatar utente
414N
Iper Master
Iper Master
Messaggi: 2922
Iscritto il: mer 13 feb 2008, 16:19
Slackware: 15.0
Kernel: 5.15.19
Desktop: KDE5
Località: Bulagna
Contatta:

Re: Ubuntu 12.10 Vs Ati catalist 12.6

Messaggio da 414N »

Basterebbe leggere attentamente gli output dei comandi:

Codice: Seleziona tutto

Module build for the currently running kernel was skipped since the
kernel source for this kernel does not seem to be installed.
ovvero: "la compilazione del modulo per il kernel attualmente in esecuzione è stata saltata dato che i sorgenti di questo kernel non sembrano installati".
Direi che la cura, in questo caso, consiste nell'installare i sorgenti del kernel attuale (ci sarà un qualche pacchetto apposito nel repository), il meta-pacchetto "build-essential" (così poi la compilazione dovrebbe filare liscia senza altre sorprese) e reinstallare i pacchetti legacy provenienti da quel repository esterno perché, come già detto, è inutile installare gli ultimi driver dato che non supportano la tua scheda...

DarkNeo
Linux 4.x
Linux 4.x
Messaggi: 1071
Iscritto il: sab 4 giu 2005, 0:00
Nome Cognome: Stefano Arbitrio
Slackware: Slackware64 14
Kernel: 3.2.X
Desktop: KDE
Località: Matrix

Re: Ubuntu 12.10 Vs Ati catalist 12.6

Messaggio da DarkNeo »

Sono sempre i driver 12.6 ma a prescindere da come li compilo sia seguendo la guida che in altri modi non funzionano resta il fatto che non mi trova il modulo fglrx da caricare

Avatar utente
414N
Iper Master
Iper Master
Messaggi: 2922
Iscritto il: mer 13 feb 2008, 16:19
Slackware: 15.0
Kernel: 5.15.19
Desktop: KDE5
Località: Bulagna
Contatta:

Re: Ubuntu 12.10 Vs Ati catalist 12.6

Messaggio da 414N »

Ti chiedo scusa, ho travisato la versione dei driver nel tuo precedente post.
Resta il fatto che ti convenga comunque utilizzare quel repository esterno per l'installazione.
Come ti ho appena spiegato non hai compilato proprio nulla, dato che non hai installati i sorgenti del kernel o, se sono installati, non vengono trovati dalla procedura di installazione dei driver AMD.
Installa i sorgenti del kernel (un "apt-get install linux-source" dovrebbe bastare) e vedrai che il modulo fglrx verrà compilato (si spera :)).

DarkNeo
Linux 4.x
Linux 4.x
Messaggi: 1071
Iscritto il: sab 4 giu 2005, 0:00
Nome Cognome: Stefano Arbitrio
Slackware: Slackware64 14
Kernel: 3.2.X
Desktop: KDE
Località: Matrix

Re: Ubuntu 12.10 Vs Ati catalist 12.6

Messaggio da DarkNeo »

Sto reinstallando ubuntu sulla partizione al fine di escludere ogni possibile errore commesso in precedenza. Riproverò come mi hai consigliato. Ma per curiosità cose Linux-headers-generic?

Avatar utente
414N
Iper Master
Iper Master
Messaggi: 2922
Iscritto il: mer 13 feb 2008, 16:19
Slackware: 15.0
Kernel: 5.15.19
Desktop: KDE5
Località: Bulagna
Contatta:

Re: Ubuntu 12.10 Vs Ati catalist 12.6

Messaggio da 414N »

Penso sia il pacchetto contenente i soli header-files del kernel. Te l'ha suggerito apt-get?
I sorgenti del kernel vengono spesso divisi in sorgenti veri e propri (includono gli header e tutto il codice implementativo) e soli header (non hanno il codice implementativo ma solo le dichiarazioni delle funzioni, variabili ecc.). Questa separazione viene spesso effettuata perché la dimensione dei sorgenti in-toto non è trascurabile e le applicazioni di sistema, solitamente, non necessitano d'altro se non degli header del kernel per effettuare chiamate di sistema (syscall).
Nel caso di moduli del kernel esterni (come fglrx), invece, diventa necessario avere tutti i sorgenti sottomano (includendo la configurazione del kernel attuale) perché non si sta compilando un'applicazione che si "appoggia" alle funzionalità fornite del kernel ma, piuttosto, un nuovo componente del kernel stesso da "incastrare" in mezzo agli altri componenti già presenti.

Rispondi